Pharm3 Final Exam

¡Supera tus tareas y exámenes ahora con Quizwiz!

8. 8.ID: 10068756864 The nurse has a prescription to administer vasopressin (Pitressin) to a pediatric patient for gastrointestinal hemorrhage. The dose prescribed is 0.003 units/kg/min. The patient's weight is 55 lb. What is the correct infusion rate for vasopressin?

0.075 units/min The patient's weight of 55 lb is converted to kilograms by dividing 55 by 2.2, which equals 25 kg. When 25 kg is multiplied by 0.003 units/kg/min, the result is 0.075 units/min.

6 The order reads: Give hydralazine (Apresoline) 0.75 mg/kg/day. The child weighs 16 pounds. How much hydralazine will be given? Round to hundredths.

6. 5.45 mg/kg/day

6 The nurse is administering an intravenous infusion of a phosphodiesterase inhibitor to a patient who has heart failure. The nurse will evaluate the patient for which therapeutic effects? (Select all that apply.) a Positive inotropic effects b Vasodilation c Decreased heart rate d Increased blood pressure e Positive chronotropic effects

6. a, b, e a Positive inotropic effects b Vasodilation e Positive chronotropic effects

8. 8.ID: 10068753442 The nurse would question the use of milrinone (Primacor) in a patient with which condition? A. Acute renal failure B. Aortic regurgitation C. Systolic heart failure D. Mitral valve prolapse

B. Aortic regurgitation Milrinone (Primacor), a PDI, is contraindicated in severe aortic or pulmonary valvular disease and in diastolic heart failure.

7. 7.ID: 10068752042 The nurse would teach a client prescribed metronidazole (Flagyl) to avoid ingestion of which drink? A. Milk B. Wine C. Coffee D. Orange juice

B. Wine A disulfiram-like (Antabuse) reaction may occur with concurrent ingestion of metronidazole and alcohol, leading to facial flushing, tachycardia, palpitations, nausea, and vomiting.

2. 2.ID: 10068762590 A patient receiving phenytoin (Dilantin) has a serum drug level drawn. Which level will the nurse note as therapeutic? A. 8 mcg/mL B. 30 mcg/mL C. 12 mcg/mL D. 6 mcg/mL

C. 12 mcg/mL Therapeutic serum drug levels for phenytoin are between 10 and 20 mcg/mL.

8. 8.ID: 10068741279 The nurse is providing care for a patient with a diagnosis of Parkinson's disease who has preexisting liver failure. What class of medications, if prescribed, would the nurse question? A. Anticholinergics B. Ergot derivative C. Dopamine modulator D. Catechol ortho-methyltransferase (COMT) inhibitors

D. Catechol ortho-methyltransferase (COMT) inhibitors COMT inhibitors are contraindicated in patients with liver failure or liver disease. Potential liver failure is also a known adverse effect.

3 Which statement by the patient reflects the need for additional patient education about the calcium channel blocker diltiazem (Cardizem)? a "I can take this drug to stop an attack of angina." b "I understand that food and antacids alter the absorption of this oral drug." c "When the long-acting forms are taken, the drug cannot be crushed." d "This drug may cause my blood pressure to drop, so I need to be careful when getting up."

a "I can take this drug to stop an attack of angina."

4 The nurse administering a cholinergic-blocking drug to a patient who is experiencing drug-induced extrapyramidal effects would assess for which therapeutic effect? a Decreased muscle rigidity and tremors b Increased heart rate c Decreased bronchial secretions d Decreased GI motility and peristalsis

a Decreased muscle rigidity and tremors

2 After a nebulizer treatment with the beta agonist albuterol, the patient complains of feeling a little "shaky," with slight tremors of the hands. The patient's heart rate is 98 beats/min, increased from the pretreatment rate of 88 beats/min. The nurse knows that this reaction is an a expected adverse effect of the medication. b allergic reaction to the medication. c indication that he has received an overdose of the medication. d idiosyncratic reaction to the medication.

a expected adverse effect of the medication.

1 A patient is experiencing severe diarrhea, flushing, and life-threatening hypotension associated with carcinoid crisis. The nurse will prepare to administer which drug? a octreotide (Sandostatin) b vasopressin (Pitressin) c somatropin (Humatrope) d cosyntropin (Cortrosyn)

a octreotide (Sandostatin)

1 In caring for a patient experiencing ethanol withdrawal, the nurse expects to administer which medication or medication class as treatment for this condition? a lithium (Eskalith) b Benzodiazepines c buspirone (BuSpar) d Antidepressants

b Benzodiazepines

2 During a teaching session for a patient who is receiving inhaled salmeterol, the nurse emphasizes that the drug is indicated for which condition? a Rescue treatment of acute bronchospasms b Prevention of bronchospasms c Reduction of airway inflammation d Long-term treatment of sinus congestion

b Prevention of bronchospasms

5 After the patient takes a dose of an inhaled corticosteroid, such as fluticasone (Flovent), what is the most important action the patient needs to do next? a Hold the breath for 60 seconds. b Rinse out the mouth with water. c Follow the corticosteroid with a bronchodilator inhaler, if ordered. d Repeat the dose in 15 minutes if the patient feels short of breath.

b Rinse out the mouth with water.

2 The nurse is reviewing the drugs currently taken by a patient who will be starting drug therapy with carbamazepine (Tegretol). Which drug may raise a concern for interactions? a digoxin (Lanoxin) b acetaminophen (Tylenol) c diazepam (Valium) d warfarin (Coumadin)

b acetaminophen (Tylenol)

1 When monitoring the laboratory values of a patient who is taking antithyroid drugs, the nurse knows to watch for a increased platelet counts. b decreased white blood cell counts. c decreased blood urea nitrogen level. d increased blood glucose levels.

b decreased white blood cell counts.

3 When assessing a patient who is about to receive an albumin infusion, the nurse knows that a contraindication for albumin would be a acute liver failure. b heart failure. c severe burns. d fluid-volume deficit.

b heart failure.

1 Which condition will alert the nurse to a potential caution or contraindication regarding the use of a dopaminergic drug for treatment of mild Parkinson's disease? a Diarrhea b Tremors c Angle-closure glaucoma d Unstable gait

c Angle-closure glaucoma

2 When assessing a patient who has been taking amiodarone for 6 months, the nurse monitors for which potential adverse effect? a Hyperglycemia b Dysphagia c Photophobia d Urticaria

c Photophobia

5 Before initiating therapy with a nonselective beta blocker, the nurse will assess the patient for a history of which condition? a Hypertension b Liver disease c Pancreatitis d Asthma

d Asthma

3 When assessing the elderly patient, the nurse keeps in mind that certain nonspecific symptoms may represent hypothyroidism in these patients, such as: a leukopenia, anemia b loss of appetite, polyuria c weight loss, dry cough d cold intolerance, depression

d cold intolerance, depression

8. 8.ID: 10068753478 Which statement by the patient indicates the need for additional teaching on phenothiazine (Thorazine) drug therapy? A. "I need to change positions slowly to prevent dizziness." B. "I will call my health care provider for abnormal tongue movements" C. "I will need to wear sunscreen and protective clothing when outdoors." D. "It is okay to take this drug with a small glass of wine to help relax me."

"It is okay to take this drug with a small glass of wine to help relax me." Drinking alcohol with phenothiazines puts the patient at risk for increased central nervous system depression.

9. 9.ID: 10068745709 The health care provider has prescribed to administer IV atropine 0.02 mg/kg to a max of 0.5 mg for the treatment of bradycardia. The nurse assess that the patient has a heart rate of 32 beats/min. If the patient's weight is 78 lb, what dose of atropine will the nurse administer?

0.5 mg The patient's weight of 78 lb is converted to kilograms by dividing 78 by 2.2, which equals 35.45 kg. When 35.45 kg is multiplied by 0.02 mg/kg, the result is 0.709 mg. Because 0.709 mg exceeds the maximum dose of 0.5 mg, administer only 0.5 mg.

8. 8.ID: 10068745751 The nurse is providing care for a patient with a tricyclic antidepressant overdose. The health care provider prescribes physostigmine (Antilirium) intravenously at 0.02 mg/kg repeated every 5 to 10 minutes to achieve the desired effect for the patient. The patient weighs 92 lb. What dose amount will the nurse administer every 5 to 10 minutes?

1. 0.84 mg The patient's weight of 92 lb is converted to kilograms by dividing 92 by 2.2, which equals 41.82 kg. When 41.82 kg is multiplied by 0.02 mg/kg, the result is 0.84 mg.

8. 8.ID: 10068756824 The nurse has a prescription to infuse dobutamine (Dobutrex) intravenously at 3 mcg/kg/min in a pediatric patient with cardiac decompensation. The patient's weight is 77 lb. What is the correct dosage to administer?

1. 105 mcg/min The patient's weight of 77 lb is converted to kilograms by dividing 77 by 2.2, which equals 35 kg. 3 mcg/kg/min multiplied by 35 kg equals 105 mcg/min.

10. 10.ID: 10068753428 A pediatric patient weighing 66 lb is prescribed digoxin (Lanoxin) 12 mcg/kg in 3 evenly divided doses. How much will the nurse administer per dose?

1. 120 mcg To identify the dosage: 66 lb divided by 2.2 = 30 kg × 12 mcg/kg = 360 mcg/3 doses = 120 mcg/dose.

11. 11.ID: 10068761796 The nurse is caring for a pediatric patient with a prescription for 0.2 mg/kg bolus of verapamil (Calan). What is the correct dose for a pediatric patient with a weight of 34 lb?

1. 3.1 mg The pediatric patient's weight of 34 lb is converted to kilograms by dividing 34 by 2.2, which equals 15.45 kg. When 15.45 kg is multiplied by 0.2 mg/kg, the result is 3.09 mg, which rounds to 3.1 mg.

8. 8.ID: 10068758198 The nurse has a prescription to change the patient's levothyroxine (Synthroid) dosage from oral to intravenous (IV). The patient takes 150 mcg/day by mouth, and the prescription is to administer 50% of the oral dose by the IV route. How much will the nurse administer per day?

1. 75 mcg 150 mcg/day × 0.50 = 75 mcg

8. 8.ID: 10068753416 The nurse has a prescription to administer 500 mL of lactated Ringer's solution over 4 hours. What is the correct pump setting for this prescription?

125 mL/hr 500 mL divided by 4 hours equals 125 mL/hr.

8. 8.ID: 10068745095 A pediatric patient with neurogenic bladder has a prescription to administer oxybutynin (Ditropan) 0.2 mg/kg/dose. The patient's weight is 32 lb. The nurse would administer what dose?

2.9 mg The patient's weight of 32 lb is converted to kilograms by dividing 32 by 2.2, which equals 14.55 kg. When 14.55 kg is multiplied by 0.2 mg/kg, the result is 2.9 mg per dose.

6 The nurse is administering an intravenous dose of morphine sulfate to a 48-year-old postoperative patient. The dose ordered is 3 mg every 3 hours as needed for pain. The medication is supplied in vials of 4 mg/mL. How much will be drawn into the syringe for this dose?

6. 0.75 mL

6 The nurse is administering an NMBD to a patient during a surgical procedure. Number the following phases of muscle paralysis in the order in which the patient will experience them. (Number 1 is the first step.) a Paralysis of intercostals and diaphragm muscles b Muscle weakness c Paralysis of muscles of the limbs, neck, and trunk d Paralysis of small rapidly moving muscles (fingers, eye)

6. a = 4, b = 1, c = 3, d = 2 b Muscle weakness d Paralysis of small rapidly moving muscles (fingers, eye) c Paralysis of muscles of the limbs, neck, and trunk a Paralysis of intercostals and diaphragm muscles

9. 9.ID: 10068753402 The nurse has a prescription to administer 250 mL of half-normal saline (0.45%NS) over 4 hours. What is the correct pump setting for this prescription?

63 mL/hr 250 mL divided by 4 hours equals 62.5 mL/hr, which rounds to 63 mL/hr.

7 The medication order for a 5-year-old child reads: "Give digoxin elixir, 15 mcg/kg, PO now." The child weighs 20 kg. How many milligrams will this child receive?

7. 0.3 mg

7 The order reads: "Give ocreotide (Sandostatin) 50 mcg subcut twice a day." The medication is available in an injectable form of 0.05 mg/mL. How many milliliters will the nurse draw up for the ordered dose?

7. 1 mL

7. A patient who has had an accidental overdose of tricyclic antidepressants is to receive physostigmine (Antilirium), 1.5 mg IM stat. The medication is available in a vial that contains 2 mL, with a concentration of 1 mg/mL. How much medication will the nurse draw up into the syringe for this dose?

7. 1.5 mL

7 The preoperative order for an adult patient reads: "Give scopolamine, 0.7 mg IM on call for surgery." The medication is available in vials of 0.4 mg/mL. How many milliliters will the nurse administer for this dose? (Round to tenths.)

7. 1.8 mL

7 The nurse is giving an intravenous dose of levothyroxine (Synthroid). The order reads: "Give 0.1 mg IV push now." What is the ordered dose in micrograms?

7. 100 mcg

7 The order reads: "Infuse 1000 mL of normal saline over the next 8 hours." The IV tubing has a drop factor of 15 gtt/mL. Calculate the mL/hour rate, and calculate the drops/minute setting for the IV tubing with this gravity infusion.

7. 125 mL/hr; 31 gtt/min

7 The medication order reads: niacin, 500 mg PO, every evening. The medication is available in 250-mg tablets. How many tablets will the patient receive per dose?

7. 2 tablets

7 A patient is in the emergency department with new-onset rapid rate atrial fibrillation. The nurse is about to add a continuous infusion of diltiazem (Cardizem) at 5 mg/hr, but must first give a bolus of 0.25 mg/kg over 2 minutes. The patient weighs 220 pounds. The medication comes in a vial of 5 mg/mL. How many milligrams will the patient receive, and how many milliliters will the nurse draw up for this dose?

7. 25 mg; 5 mL

7. The order reads, "Give metoprolol (Lopressor) 300 mg/day PO in 2 divided doses." The tablets are available in 50-mg strength. How many tablets will the patient receive per dose?

7. 3 tablets per dose (150 mg per dose)

7 A child in the pediatric ICU has been receiving a dopamine infusion. This morning while on rounds, the nurse noted that the IV has infiltrated. After stopping the infusion, the nurse prepares to administer phentolamine (Regitine). The ordered dose is 0.2 mg/kg, to be injected into the area of extravasation. The child weighs 39 lb. How many milligrams will the nurse administer? (Round to tenths.)

7. 3.5 mg

7 The order reads: Give mannitol 0.5 g/kg IV now, over 2 hours. The patient weighs 165 lb, and you have a 100-mL vial of 20% mannitol. How many grams will the patient receive? How many milliliters of mannitol will you prepare for this infusion?

7. 37.5 g, 187.5 mL

7 A patient with a feeding tube will be receiving risperidone (Risperdal) 8 mg in 2 divided doses via the feeding tube. The medication is available in a 1 mg/mL solution. How many milliliters will the nurse administer for each dose?

7. 4 mL per dose

7 The order reads: bromocriptine (Parlodel) 10 mg per day PO. The medication is available in 2.5-mg tablets. How many tablets will the nurse give per dose?

7. 4 tablets

7 The order for a child reads: "Give cefoxitin (Mefoxin) 160 mg/kg/day, IVPB, divided into doses given every 6 hours." The child weighs 55 lb. How much will the patient receive each day? For each dose?

7. 4000 mg/day; 1000 mg/dose

7 A patient comes to the clinic complaining of elbow pain after an injury. He states that he has been taking two pain pills, eight times a day, for the past few days. The medication bottle contains acetaminophen, 325-mg tablets. Calculate how much medication he has been taking per day. Is this a safe dose of this medication?

7. 5200 mg/day, No

7 A child is to receive phenobarbital 2 mg/kg IV on call as a preoperative sedative. The child weighs 64 pounds. How many milligrams will the child receive for this dose?

7. 58.2 mg

7 The nurse is preparing to administer valproic acid to a child. The order reads: "Give valproic acid, 15 mg/kg/day PO in three divided doses." The child weighs 33 pounds. How many milligrams will the child receive with each dose?

7. 75 mg per dose

7 The order reads: "Dopamine 3 mcg/kg/min IV." The solution available is 400 mg in 250 mL D5W, and the patient weighs 176 pounds. The nurse will set the IV infusion pump to run at how many mL/hour?

7. 9 mL/hr

7 A patient has been given an MDI of albuterol and is instructed to take two puffs three times a day, with doses 6 hours apart. The inhaler contains 200 actuations, but does not have a dose counter. Calculate how many days the inhaler will deliver this ordered dose.

7. Approximately 33 days (6 puffs/day divided into 200)

7. 7.ID: 10068756878 The health care provider prescribes somatropin (Genotropin) in a dosage of 0.35 mg/kg/wk intramuscularly for growth hormone deficiency. The patient's weight is 48 lb. What is the correct dose for the nurse to administer?

7.6 mg/wk The patient's weight of 48 lb is converted to kilograms by dividing 48 by 2.2, which equals 21.81 kg. When 21.81 kg is multiplied by 0.35 mg/kg/week, the result is 7.63 mg, which rounds to 7.6 mg.

1. 1.ID: 10068749233 The nurse provides discharge instructions to a patient prescribed cholestyramine (Questran). Which statement by the patient indicates teaching was effective for this drug? A. "I will increase fiber in my diet and drink more fluids." B. "I will notify my health care provider if I have muscle pain." C. "I will take Questran 1 hour before my other medications." D. "This drug can cause flushing, itching and gastrointestinal upset."

A. "I will increase fiber in my diet and drink more fluids." Correct Cholestyramine can cause constipation; thus increasing dietary fiber and fluid intake is appropriate. All other drugs should be taken 1 hour before or 4 hours after cholestyramine to facilitate proper absorption.

8. 8.ID: 10068745087 Which statement by a client best indicates an understanding of the teaching on flunisolide (AeroBid)? A. "I will rinse my mouth with water after each use." B. "I will wash the plastic inhaler casing once a month." C. "I will take two puffs to treat an acute asthma attack." D. "I will not use my albuterol inhaler while I am taking AeroBid."

A. "I will rinse my mouth with water after each use." Flunisolide is an inhaled corticosteroid. Rinsing the mouth immediately after each use of the inhaler or nebulizer will help prevent oral candidal infections. It is not used to treat an acute asthma attack and should be taken with the client's bronchodilator medications. The plastic inhaler casing is washed in warm, soapy water every week.

2. 2.ID: 10068746653 The patient states to the nurse, "My friend said nitroglycerin relieves angina pain by reducing preload. What is preload?" Which statement by the nurse explains preload to this patient? A. "It is the blood return to the heart." B. "It is the oxygen demand of the heart." C. "It is the pressure against which the heart must pump." D. "It is dilation of arteries and veins throughout the body."

A. "It is the blood return to the heart." Correct Preload is determined by the amount of blood in the ventricle just before contraction.

4. 4.ID: 10068753436 A patient weighing 44 lb is prescribed a digoxin (Lanoxin) loading dose of 0.03 mg/kg to be administered in three divided doses. How much will the nurse administer in each dose? A. 0.2 mg B. 0.3 mg C. 0.4 mg D. 0.6 mg

A. 0.2 mg 44 lb is converted to kilograms by dividing 44 by 2.2 kg = 20 kg. 0.03 mg/kg × 20 kg = 0.6 mg. 0.6 mg ÷ 3 doses = 0.2 mg/dose.

6. 6.ID: 10068752056 The nurse is planning care for a client prescribed once-daily IV gentamicin (Garamycin) therapy. When should the nurse schedule a trough drug level to be drawn? A. 12 hours after completing the antibiotic infusion B. 18 hours after completing the antibiotic infusion C. 30 minutes after beginning the antibiotic infusion D. 60 minutes after beginning the antibiotic infusion

A. 12 hours after completing the antibiotic infusion Trough serum drug levels should be drawn at least 8 to 12 hours after the medication is infused.

4. 4.ID: 10068756832 When assessing for cardiovascular effects of an adrenergic (sympathomimetic) drug, the nurse understands that these drugs produce which effect on the heart? A. A positive inotropic, positive chronotropic, and positive dromotropic effect B. A negative inotropic, positive chronotropic, and positive dromotropic effect C. A positive inotropic, negative chronotropic, and negative dromotropic effect D. A negative inotropic, negative chronotropic, and negative dromotropic effect

A. A positive inotropic, positive chronotropic, and positive dromotropic effect Adrenergic stimulation of the beta1-adrenergic receptors on the myocardium and in the conduction system of the heart results in an increased heart rate (positive chronotropic effect), increased contractility (positive inotropic effect), and increased conductivity (positive dromotropic effect).

2. 2.ID: 10068762508 The nurse is reviewing the protocol for administration of IV adenosine (Adenocard). What is the MOST important nursing intervention to remember when giving this medication? A. Administer it as a fast IV push. B. Assess for burning sensation at the IV site. C. Monitor the infusion site for hematoma. D. Flush the IV catheter with normal saline.

A. Administer it as a fast IV push. Correct Adenosine must be given as rapidly as possible because it has an extremely short half-life of less than 10 seconds. For this reason, it is administered only IV and only as a fast IV push.

1. 1.ID: 10068758882 What is the priority assessment data for a client prescribed antibiotic therapy? A. Allergies B. Immunizations C. History of seizures D. Cardiac dysrhythmias

A. Allergies Antibiotic allergy is one of the most common drug allergies. An allergic reaction that occurs after administration of an antibiotic has the potential to cause severe anaphylaxis and possible death.

An intravenous dose of midazolam (Versed) has been prescribed for a patient before a colonoscopy. The nurse informs the patient that one of the most common adverse effects of this medication is what effect? A. Amnesia B. Dry mouth C. Chest pain D. Constipation

A. Amnesia Versed is known to cause amnesia and anxiolysis (reduced anxiety) as well as sedation and is most commonly used preoperatively for certain procedures.

13. 13.ID: 10068758824 A patient prescribed spironolactone (Aldactone) asks the nurse to assist with food choices that are low in potassium. The nurse would recommend which food choices? (Select all that apply.) A. Apples Correct B. Bananas C. Pineapple Correct D. Lean meat Correct E. Winter squash

A. Apples Correct C. Pineapple Correct D. Lean meat Correct Spironolactone is a potassium-sparing diuretic that could potentially cause hyperkalemia. Bananas and winter (not summer) squash are high in potassium and should be avoided in patients taking spironolactone.

3. 3.ID: 10068762502 To prevent the occurrence of cinchonism in a patient prescribed quinidine (Quindex), which instruction is MOST important for the nurse to provide for this patient? A. Avoid drinking grapefruit juice. B. Remind the patient to change positions slowly. C. Advise the patient to wear sunscreen every day. D. Increase dietary intake of potassium.

A. Avoid drinking grapefruit juice. Correct Grapefruit juice can inhibit the metabolism of quinidine, which increases the risk of cinchonism.

5. 5.ID: 10068753472 A patient with a diagnosis of depression is being discharged with a prescription for an MAOI. Which instruction should the nurse include for this medication? A. Avoid eating aged cheese. B. Encourage use of fiber supplements. C. Explain the symptoms of tardive dyskinesia. D. Emphasize that tremors are a common adverse effect.

A. Avoid eating aged cheese. Eating foods high in tyramine, including aged cheese, can cause a hypertensive crisis in patients taking MAOIs.

1. 1.ID: 10068753482 When doing an admission drug history, the nurse notes that the patient has a prescription for lithium (Lithobid). The nurse suspects that this patient has been diagnosed with which condition? A. Bipolar disorder B. Absence seizures C. Paranoid schizophrenia D. Obsessive-compulsive disorder

A. Bipolar disorder Correct Lithium is a mood-stabilizing drug for the treatment of manic episodes associated with bipolar disorders.

9. 9.ID: 10068743961 The nurse is caring for a patient with a history of angina and hypertension who is diagnosed with moderate heart failure. The patient's current medication regimen includes digoxin (Lanoxin), furosemide (Lasix), and quinapril (Accupril). Which medication would be most beneficial for the health care provider to add to this patient's treatment plan? A. Carvedilol (Coreg) B. Sotalol (Betapace) C. Esmolol (Brevibloc) D. Propranolol (Inderal)

A. Carvedilol (Coreg) Correct Carvedilol (Coreg), a combined alpha11 and beta blocker, has been shown to slow the progression of heart failure and decrease the frequency of hospitalization in patients with mild to moderate (Class II or III) heart failure. Carvedilol is most commonly added to digoxin, furosemide, and angiotensin-converting enzyme inhibitors when used to treat heart failure.

7. 7.ID: 10068762500 Before administering a dose of an antidysrhythmic drug to a patient, what is the priority nursing assessment? A. Check apical pulse and blood pressure. B. Measure urine output and specific gravity. C. Obtain temperature and pulse oximetry on room air. D. Evaluate peripheral pulses and level of consciousness.

A. Check apical pulse and blood pressure. Correct Antidysrhythmic drugs can cause both hypotension and bradycardia; therefore, it is important to assess blood pressure and apical pulse before administration.

17. 17.ID: 10068746657 For a patient receiving IV nitroglycerin (Tridil), what are the priority nursing interventions? (Select all that apply.) A. Check the heart rate. B. Monitor blood pressure. C. Auscultate lung sounds. D. Measure intake and output. E. Assess for worsening chest pain.

A. Check the heart rate. Correct B. Monitor blood pressure. Correct E. Assess for worsening chest pain. Correct IV nitroglycerin can cause sudden and severe hypotension, worsening of chest pain, and significant changes in heart rate (less than 60 beats/min or greater than 100 beats/min).

5. 5.ID: 10068762510 Sodium channel blockers are considered which class of antidysrhythmic drugs? A. Class I B. Class II C. Class III D. Class IV

A. Class I The Vaughan Williams classification is the most commonly used system to classify antidysrhythmic drugs. There are four major classes of antidysrhythmic drugs: I, II, III, and IV. Sodium channel blockers are classified as class I drugs.

12. 12.ID: 10068758890 Bacterial resistance to antibiotics can occur with which situations? (Select all that apply.) A. Clients stop taking an antibiotic when they feel better. B. Antibiotics that are prescribed to treat a viral infection C. Taking an antibiotic and an antiviral medication at the same time D. Microorganisms arriving from foreign countries and overseas ports E. Antibiotics that are prescribed according to culture and sensitivity reports

A. Clients stop taking an antibiotic when they feel better. B. Antibiotics that are prescribed to treat a viral infection Not completing a full course of antibiotic therapy can allow bacteria that are not killed but have been exposed to the antibiotic to adapt their physiology to become resistant to that antibiotic. Administering antibiotics to treat viral infections is not effective and may expose small amounts of bacteria that may be present to the antibiotic and therefore risk the development of resistance.

11. 11.ID: 10068752062 When providing instructions to clients on use of antibiotics, which instructions would the nurse include in the teaching? (Select all that apply.) A. Complete the entire course of therapy. B. Increase fluid intake up to 3000 mL/day. C. Wash your hands before and after preparing food. D. Notify the provider of any possible reactions that occur. E. Save unused medication in a cool dry place for later use.

A. Complete the entire course of therapy. B. Increase fluid intake up to 3000 mL/day. C. Wash your hands before and after preparing food. D. Notify the provider of any possible reactions that occur. There should not be any leftover medication, but if there is, it needs to be discarded in the appropriate method. The health care provider typically only writes a prescription for the exact amount of medication needed by the client.

6. 6.ID: 10068762580 The nurse is caring for a postoperative craniotomy patient. Which prescribed drug does the nurse make sure is readily available to treat acute seizure activity? A. Diazepam (Valium) B. Gabapentin (Neurontin) C. Ethosuximide (Zarontin) D. Flumazenil (Romazicon)

A. Diazepam (Valium) Drug therapy for acute seizure activity is typically diazepam (Valium), which is considered by many to be the drug of choice. Other drugs used for acute therapy include lorazepam, fosphenytoin, phenytoin, and phenobarbital.

15. 15.ID: 10068758814 Potassium-sparing diuretics may cause which common adverse reactions? (Select all that apply.) A. Dizziness B. Headache C. Hyperkalemia D. Mental confusion E. Muscle weakness

A. Dizziness Correct B. Headache Correct C. Hyperkalemia Correct Hyperkalemia, dizziness, and headache are common adverse effects associated with potassium-sparing diuretics.

5. 5.ID: 10068758888 When planning care for a client receiving a sulfonamide antibiotic, it is important for the nurse to perform which intervention? A. Encourage fluid intake of 2000 to 3000 mL/day. B. Avoid direct sun exposure and tanning beds. C. Take the medication with dairy products such as milk or yogurt. D. Advise the client to report any tinnitus to the health care provider.

A. Encourage fluid intake of 2000 to 3000 mL/day. Clients should be encouraged to drink plenty of fluids (2000 to 3000 mL/24 hours) to prevent drug-related crystalluria associated with sulfonamide antibiotics.

16. 16.ID: 10068746669 The nurse understands that a patient receiving nitroglycerin should be monitored for which common adverse effects associated with this medication? (Select all that apply.) A. Flushing B. Headache C. Dizziness D. Hypotension E. Blurred vision

A. Flushing Correct B. Headache Correct C. Dizziness Correct D. Hypotension Correct The common adverse effects of nitroglycerin include flushing of the face, dizziness, fainting, headache, and hypotension.

9. 9.ID: 10068758810 To treat a patient with pulmonary edema, the nurse prepares to administer which diuretic to this patient? A. Furosemide (Lasix) B. Amiloride (Midamor) C. Triamterene (Dyrenium) D. Spironolactone (Aldactone)

A. Furosemide (Lasix) Correct Furosemide is a potent, rapid-acting diuretic that would be the drug of choice to treat pulmonary edema. The other medications are not potent enough to cause the diuresis necessary to treat this condition.

12. 12.ID: 10068749247 A patient with elevated triglyceride levels unresponsive to HMG-CoA reductase inhibitors will most likely be prescribed which drug? A. Gemfibrozil (Lopid) B. Cholestyramine (Questran) C. Colestipol (Colestid) D. Simvastatin (Zocor)

A. Gemfibrozil (Lopid) Correct Gemfibrozil, a fibric acid derivative, promotes catabolism of triglyceride-rich lipoproteins.

5. 5.ID: 10068756876 After administering somatropin (Genotropin) to a patient, the nurse would assess for potential adverse effects of this medication by monitoring which laboratory test result? A. Glucose B. Platelets C. Potassium D. Magnesium

A. Glucose Hyperglycemia and hypoglycemia are potential adverse effects of somatropin therapy.

Which descriptions apply to the market for medicinal herbs in the United States? (Select all that apply.) A. Growing in numbers of users B. Used with traditional therapies C. Less than $1 million in sales annually D. Used by less than 40% of the population E. Viewed with growing skepticism F. Heavily regulated by federal law

A. Growing in numbers of users Correct B. Used with traditional therapies Correct The market for herbal remedies is growing, with over-the-counter (OTC) medications now accounting for about 60% of all medications used in the United States. Herbal remedies may be used in traditional therapies. It is a largely unregulated market, generating billions of dollars in sales annually. There is a growing market of $20 billion in the sales of herbs OTC with fewer controversies about safety and control of the market. The FDA does not regulate these products unless there are insufficient data.

4. 4.ID: 10068745757 What is an adverse effect of bethanechol (Urecholine)? A. Headache B. Tachypnea C. Constipation D. Hypertension

A. Headache Correct Adverse effects include syncope, hypotension with reflex tachycardia, headache, seizure, GI upset, and asthma attacks.

10. 10.ID: 10068741265 A patient is prescribed selegiline (Eldepryl), an MAO-B inhibitor, as adjunctive therapy in treatment of Parkinson's disease. What potential adverse effects should the nurse include in education for this patient? (Select all that apply.) A. Headache B. Insomnia C. Weight gain D. Depression E. Blood pressure changes

A. Headache Correct B. Insomnia Correct D. Depression Correct E. Blood pressure changes Correct MAO-B inhibitors have many potential adverse effects, including dizziness, insomnia, hallucinations, ataxia, agitation, depression, paresthesia, somnolence, headache, dyskinesia, nausea, diarrhea, hypotension or hypertension, chest pain, weight loss, dermatologic reactions, rhinitis, and pharyngitis.

10. 10.ID: 10068743965 Beta blockers are used to treat which conditions? (Select all that apply.) A. Hypertension B. Angina pectoris C. Raynaud's disease D. Cardiac dysrhythmias E. COPD

A. Hypertension Correct B. Angina pectoris Correct D. Cardiac dysrhythmias Correct Beta blockers are effective in treating hypertension (secondary to negative inotropic effects), angina pectoris (decreases cardiac workload when decreasing heart rate and contractility), and cardiac dysrhythmias (decreasing heart rate and conductivity). Beta blockers can exacerbate COPD because they block beta22 receptors and subsequent bronchodilation. Raynaud's disease may be treated with alpha, not beta, blockers.

3. 3.ID: 10068758816 Which laboratory test result is a common adverse effect of furosemide (Lasix)? A. Hypokalemia B. Hypernatremia C. Hyperchloremia D. Hypophosphatemia

A. Hypokalemia Correct Furosemide is a potent loop diuretic, and the most common adverse effect of loop diuretics is electrolyte imbalances. This results in major electrolyte losses of potassium and sodium and, to a lesser extent, calcium.

6. 6.ID: 10068758892 When administering a nonsteroidal antiinflammatory drug and a penicillin drug together, the displacement of the penicillin antibiotic from the protein-binding sites will result in which effect? A. Increased free drug in blood B. Decreased free drug in blood C. No change in free drug in blood D. Absence of free drug in the blood

A. Increased free drug in blood Drugs that are not bound to protein are free and thus active to exert their therapeutic (or toxic, if too much free) effect.

6. 6.ID: 10068745761 When providing teaching to a patient diagnosed with myasthenia gravis, which instruction regarding the administration of physostigmine (Antilirium) is MOST appropriate? A. Instruct the patient to take the medication 30 minutes before meals. B. Tell the patient to increase fluid and fiber in the diet to prevent constipation. C. Inform the patitne that common adverse effects include tachycardia and hypertension. D. Explain that if a dose is missed, the patient should double the next dose to prevent withdrawal.

A. Instruct the patient to take the medication 30 minutes before meals. Correct Drugs used for myasthenia gravis should be given about 30 minutes before meals to allow for onset of action and therapeutic effects (e.g., decreased dysphagia). Constipation, tachycardia, and hypertension are not effects of cholinergic medications. A missed dose should never be doubled.

2. 2.ID: 10068760524 Which long-acting insulin mimics natural, basal insulin with no peak action and a duration of 24 hours? A. Insulin glargine (Lantus) B. Insulin glulisine (Apidra) C. Regular insulin (Humulin R) D. NPH insulin

A. Insulin glargine (Lantus) Insulin glargine has a duration of action of 24 hours with no peaks, mimicking the natural, basal insulin secretion of the pancreas.

While completing preoperative patient teaching, which information should the nurse include for a surgery with a general anesthetic? A. It produces muscle relaxation and loss of consciousness. B. It provides moderate sedation that allows you to relax. C. It affects a specific region of the body to block pain sensation. D. It uses only one type of medication to produce sedation.

A. It produces muscle relaxation and loss of consciousness. Correct General anesthesia produces muscle relaxation (both visceral and skeletal) as well as loss of consciousness. The other answers are incorrect.

11. 11.ID: 10068745081 Which laboratory value would the nurse assess before administering zafirlukast (Accolate) to a client? A. Liver enzymes B. Cardiac enzymes C. Renal function tests D. Complete blood count

A. Liver enzymes Because use of zafirlukast may lead to liver dysfunction, liver enzyme levels should be monitored regularly, especially early in the course of therapy.

Which medication is used to treat a patient with severe adverse effects of a narcotic analgesic? A. Naloxone (Narcan) B. Acetylcysteine (Mucomyst) C. Methylprednisolone (Solu-Medrol) D. Flumazenil (Romazicon)

A. Naloxone (Narcan) Naloxone is the narcotic antagonist that will reverse the effects, both adverse and therapeutic, of opioid narcotic analgesics.

Immediate postoperative assessment of a patient reveals a body temperature of 102° F. Which action will the nurse perform? A. Notify the health care provider. B. Apply a cooling blanket per protocol. C. Assess for signs and symptoms of infection related to the surgical wound. D. Administer acetaminophen as prescribed and recheck temperature in 1 hour.

A. Notify the health care provider. A fever immediately after surgery must be evaluated for potential malignant hyperthermia. Thus, the health care provider must be notified. Infection of a surgical wound takes several days to develop.

7. 7.ID: 10068746673 Before administering isosorbide mononitrate (Imdur) sustained-release tablet to a patient, what is the priority nursing intervention? A. Obtain a blood pressure reading. B. Remind the patient to take the tablet before meals. C. Emphasize that the patient should swallow the tablet whole. D. Advise the patient that Tylenol is used to treat headache.

A. Obtain a blood pressure reading. Correct Mononitrate is a vasodilator and thus can cause hypotension. It is important to assess blood pressure before administering.

The nurse plans pharmacologic management for a patient with pain. The nurse should administer the pain medication based on what dosage schedule? A. Pain relief is best obtained by administering analgesics around the clock. B. Administer the analgesic when the pain level reaches a "6" on a scale of 1 to 10. C. Opioid analgesics should not be used for more than 24 hours to prevent drug addiction. D. Analgesics should be administered as needed (prn) to minimize adverse effects.

A. Pain relief is best obtained by administering analgesics around the clock. When pain is present for more than 12 hours a day, analgesic dosages are best administered around the clock rather than on an as-needed basis, but dosages should always be within the dosage guidelines for each drug used. The around-the-clock (or "scheduled") dosing maintains steady-state levels of the medication and prevents drug troughs and escalation of pain.

8. 8.ID: 10068752048 The client's culture has grown gram-positive cocci, and the health care provider prescribes two different antibiotics, one of which is gentamicin (Garamycin). To treat this type of infection, which type of antibiotic is typically prescribed together with gentamicin (Garamycin)? A. Penicillin B. Cephalosporin C. Fluoroquinolone D. Aminoglycoside

A. Penicillin In gram-positive cocci, gentamicin is usually given in combination with a penicillin antibiotic. The other antibiotics are not typically prescribed with gentamicin for this culture result.

7. 7.ID: 10068760536 Pramlintide (Symlin) is prescribed as supplemental drug therapy to the treatment plan for a patient with type 1 diabetes mellitus. What information should the nurse include when teaching the patient about the action of this medication? A. Pramlintide slows gastric emptying. B. Pramlintide increases glucagon excretion. C. Pramlintide stimulates glucose production. D. Pramlintide corrects insulin receptor sensitivity.

A. Pramlintide slows gastric emptying. Correct Pramlintide is a synthetic form of the naturally occurring hormone amylin. It works by slowing gastric emptying, suppressing glucagon secretion and hepatic glucose production, and increasing satiety (sense of having eaten enough). It is only administered via subcutaneously injection.

12. 12.ID: 10068745785 ACE inhibitors and ARBs both work to decrease blood pressure by which action? A. Prevent aldosterone secretion B. Increase the breakdown of bradykinin C. Enhance sodium and water resorption D. Prevent the formation of angiotensin II

A. Prevent aldosterone secretion Whereas ACE inhibitors block the formation of angiotensin II, ARBs allow the formation of angiotensin II but block its effect at the receptors. Without the receptors stimulated (because of either drug), aldosterone secretion is inhibited, preventing the reabsorption of sodium and water.

An intubated, mechanically ventilated patient in the intensive care unit (ICU) is becoming increasingly restless and anxious. The nurse expects to administer which intravenous (IV) anesthetic drug? A. Propofol (Diprivan) B. Isoflurane (Forane) C. Halothane (Fluothane) D. Nitrous oxide (Anesoxyn)

A. Propofol (Diprivan) Propofol is an IV sedative-hypnotic drug used for induction and maintenance of anesthesia as well as for sedation in patients who are intubated and mechanically ventilated in the ICU. It has a rapid onset and short duration of action, allowing for easy titration and maintenance of the patient's level of consciousness.

What is the best description of the consumer safety precautions for herbal supplements? A. Reliable data on product efficacy is scarce. B. Herbs in tamper-resistant containers are safe. C. Medicinal herbs are under strict U.S. Food and Drug Administration (FDA) control. D. The package labeling clearly states product safety.

A. Reliable data on product efficacy is scarce. Some herbal preparations are effective; many are not. A few can cause harmful or lethal effects. Herbal products are exempt from meaningful regulation. Dietary supplements do not need FDA approval before they are marketed and they require no efficacy for standards.

7. 7.ID: 10068753412 While the nurse is providing care to a patient, the health care provider prescribes an IV potassium chloride infusion. For which condition would this prescription be given? A. Renal failure B. Hypertension C. Multiple sclerosis D. Cirrhosis of the liver

A. Renal failure Correct Potassium replacement therapy is indicated in the treatment or prevention of potassium depletion in patients with severe renal disease, acute dehydration, untreated Addison's disease, severe hemolytic disease, conditions involving extensive tissue breakdown (e.g., multiple trauma, severe burns), and hyperkalemia from any cause.

14. 14.ID: 10068760520 Which actions describe the beneficial effects produced by sulfonylurea oral hypoglycemics? (Select all that apply.) A. Stimulate insulin secretion from beta cells B. Increase hepatic glucose production C. Enhance action of insulin in various tissues D. Inhibit breakdown of insulin by liver

A. Stimulate insulin secretion from beta cells C. Enhance action of insulin in various tissues D. Inhibit breakdown of insulin by liver The sulfonylureas stimulate insulin secretion from the beta cells of the pancreas; enhance the actions of insulin in muscle, liver, and adipose tissue; and prevent the liver from breaking insulin down as fast as it ordinarily would (reduced hepatic clearance). Increased hepatic glucose production would serve to increase serum glucose levels, the opposite effect of oral hypoglycemic drugs.

2. 2.ID: 10068745071 Discharge teaching to a patient receiving a beta-agonist bronchodilator should emphasize reporting which side effect? A. Tachycardia B. Nonproductive cough C. Hypoglycemia D. Sedation

A. Tachycardia Correct A beta-agonist bronchodilator stimulates the beta receptors of the sympathetic nervous system, resulting in tachycardia, bronchodilation, hyperglycemia, and increased alertness.

A mechanically ventilated patient receiving a neuromuscular-blocking drug has tearing in the eyes and increased heart rate and blood pressure. How does the nurse interpret this clinical finding? A. The patient's level of sedation is inadequate. B. The patient's response to the drug is appropriate. C. The patient is having an adverse reaction to the medication. D. The patient's dose of the neuromuscular-blocking drug is insufficient.

A. The patient's level of sedation is inadequate. Tearing in eyes and increased heart rate and blood pressure are symptoms of increased anxiety or pain. A patient receiving a neuromuscular blocking drug cannot move or communicate; thus, the nurse must rely on subtle changes to assess adequate sedation.

7. 7.ID: 10068745083 What is the role of corticosteroids in the treatment of acute respiratory disorders? A. They decrease inflammation. B. They directly dilate the bronchi. C. They stimulate the immune system. D. They increase gas exchange in the alveoli.

A. They decrease inflammation. Correct Corticosteroids can suppress the immune system. They do not directly affect bronchodilation but rather prevent bronchoconstriction as a response to inflammation.

4. 4.ID: 10068741281 What is the goal of pharmacologic therapy in treating Parkinson's disease? A. To balance cholinergic and dopaminergic activity in the brain B. To increase the amount of acetylcholine at the presynaptic neurons C. To decrease the amount of dopamine available in the substantia nigra D. To block dopamine receptors in both presynaptic and postsynaptic neurons

A. To balance cholinergic and dopaminergic activity in the brain Parkinson's disease results from a decrease in dopaminergic (inhibitory) activity, leaving an imbalance with too much cholinergic (excitatory) activity. By increasing dopamine, the neurotransmitter activity becomes more balanced and symptoms become controlled.

8. 8.ID: 10068762582 A patient receiving valproic acid (Depakote) should be monitored for which adverse effects? (Select all that apply.) A. Tremors B. Insomnia C. Weight gain D. Hepatoxicity E. Hypoglycemia

A. Tremors C. Weight gain D. Hepatoxicity Common adverse effects of valproic acid (Depakote) include drowsiness; nausea, vomiting, and other gastrointestinal disturbances; tremor; weight gain; and transient hair loss. The most serious adverse effects are hepatotoxicity and pancreatitis. It is not known to cause hypoglycemia.

3. 3.ID: 10068745749 The nurse is performing postoperative teaching for a patient prescribed bethanechol (Urecholine). The patient is prescribed this drug for the treatment of which condition? A. Urinary atony B. Ischemic colitis C. Respiratory atelectasis D. Orthostatic hypotension

A. Urinary atony Correct Bethanechol is a direct-acting cholinergic agonist that stimulates the cholinergic receptors on the smooth muscle of the bladder, leading to bladder contraction and emptying.

6. 6.ID: 10068753444 Phosphodiesterase inhibitors (PDIs) have an added advantage in treating heart failure. These drugs cause a positive inotropic effect and what other effect? A. Vasodilation B. Bronchodilation C. Vasoconstriction D. Platelet inhibition

A. Vasodilation PDIs are also called inodilators because they have both positive inotropic and vasodilatory effects.

4. 4.ID: 10068749249 Which discharge instruction should the nurse include for a patient prescribed an antilipemic medication? A. "Stop taking the medication if it causes nausea and vomiting." B. "Continue your exercise program and maintain a low-fat diet." C. "It is important to take a double dose to make up for a missed dose." D. "Lifestyle changes are no longer necessary when taking this medication."

B. "Continue your exercise program and maintain a low-fat diet." Correct Antilipemic medications are in addition to, not a replacement of, therapeutic lifestyle changes used to decrease serum cholesterol. Maintain a low-fat, low-cholesterol diet is an integrated part of a change in lifestyle.

3. 3.ID: 10068746659 Which statement by the patient demonstrates a need for further education regarding nitroglycerin (Nitrostat) sublingual tablets? A. "I should keep my nitroglycerin in a cool, dry place." B. "I can take up to four tablets at 5-minute intervals for chest pain." C. "If I get a headache, I should keep taking my nitroglycerin and use Tylenol to relieve my headache." D. "I should change positions slowly to avoid getting dizzy from the drug's effect on my blood pressure."

B. "I can take up to four tablets at 5-minute intervals for chest pain." Correct Patients are taught to take up to three sublingual tablets 5 minutes apart. According to current guidelines, if the chest pain or discomfort is not relieved in 5 minutes, after one dose, the patient (or family member) must call 911 immediately. The patient can take one more tablet while awaiting emergency care and a third tablet 5 minutes later, but no more than three tablets total. Patients should always sit or lie down before taking this medication.

8. 8.ID: 10068746675 The patient asks the nurse, "How should sublingual nitroglycerin be stored when I travel?" What is the nurse's best response? A. "You can protect it from heat by placing the bottle in an ice chest." B. "It's best to keep it in its original container away from heat and light." C. "You can put a few tablets in a resealable bag and carry it in your pocket." D. "Keep it in the glove compartment of your car to prevent exposure to heat."

B. "It's best to keep it in its original container away from heat and light." Correct Although sublingual nitroglycerin needs to be kept in a cool, dry place, it should not be placed in an ice chest, where it could freeze. It should not be kept in the glove compartment of a car and needs to be kept away from heat, not in a clear plastic bag.

4. 4.ID: 10068758188 A patient receiving propylthiouracil (PTU) asks the nurse, "How does this medication relieve symptoms?" What is the nurse's best response? A. "PTU helps your thyroid gland synthesize and use iodine, which produces hormones better." B. "PTU inhibits the formation of new thyroid hormone, thus returning your metabolism to normal." C. "PTU causes the pituitary gland to secrete thyroid-stimulating hormone, which blocks the production of hormones by the thyroid gland." D. "PTU removes thyroid hormones that are already circulating in your bloodstream, thus decreasing the adverse effects of this medication."

B. "PTU inhibits the formation of new thyroid hormone, thus returning your metabolism to normal." Correct PTU is an antithyroid medication used to treat hyperthyroidism. It works by inhibiting the synthesis of new thyroid hormone. It does not inactivate present hormone.

5. 5.ID: 10068741269 The patient asks the nurse to explain the difference between carbidopa-levodopa (Sinemet) and ropinirole (Requip). How will the nurse respond? A. "Both drugs have the same pharmacodynamic and adverse effect profiles." B. "Ropinirole is a dopamine agonist that has fewer adverse effects than carbidopa-levodopa." C. "Whereas carbidopa-levodopa acts as a dopamine agonist, ropinirole directly replaces dopamine." D. "Carbidopa-levodopa is less effective than ropinirole in treating the symptoms of Parkinson's disease."

B. "Ropinirole is a dopamine agonist that has fewer adverse effects than carbidopa-levodopa." Ropinirole stimulates specific dopamine receptors in the brain. It is more specific for the receptors associated with parkinsonian symptoms, the D2 subfamily of dopamine receptors. This in turn may have more specific antiparkinson effects, with fewer adverse effects associated with generalized dopaminergic stimulation.

9. 9.ID: 10068745079 Which statement by the nurse should be included when teaching a client about the proper use of metered-dose inhalers? A. "After you inhale the medication once, repeat until you obtain relief." B. "Wait 1 to 2 minutes before you take a second puff of the same drug." C. "Make sure that you puff out air several times after you inhale the medication." D. "Hold the inhaler in your mouth, take a deep breath, and then compress the inhaler."

B. "Wait 1 to 2 minutes before you take a second puff of the same drug." If a second puff of the same drug is ordered, instruct the client to wait 1 to 2 minutes between puffs. If a second type of inhaled drug is prescribed, instruct the client to wait 2 to 5 minutes between the medications or to take as prescribed.

9. 9.ID: 10068760528 When teaching a patient about insulin glargine (Lantus), which statement by the nurse about this drug is correct? A. "You can mix this insulin with NPH insulin to enhance its effects on glucose metabolism." B. "You cannot mix this insulin with regular insulin and thus will have to take two injections." C. "It is often combined with regular insulin to decrease the number of insulin injections per day." D. "The duration of action for this insulin is 8 to10 hours, so you will need to take it twice a day."

B. "You cannot mix this insulin with regular insulin and thus will have to take two injections." Insulin glargine is a long-acting insulin with duration of action up to 24 hours. It should not be mixed with any other insulins. It is usually dosed once daily, but it may be dosed every 12 hours depending on the patient's glycemic response.

3. 3.ID: 10068756836 A patient weighing 176 lb is to receive a dopamine (Intropin) continuous intravenous (IV) infusion at 5 mcg/kg/min. The solution strength available is dopamine 400 mg in 500 mL D5W. The nurse will infuse the medication at which rate? A. 20 mL/hr B. 30 mL/hr C. 40 mL/hr D. 50 mL/hr

B. 30 mL/hr The patient's weight of 176 lb is converted to kilograms by dividing 176 by 2.2, which equals 80 kg. 5 mcg/kg/min multiplied by 80 kg equals 400 mcg, or 0.4 mg/min. Dividing 0.4 mg/min by 400 mg/500 mL = 0.5 mL/min, which when multiplied by 60 minutes = 30 mL/hour.

6. 6.ID: 10068758836 When preparing to administer intravenous furosemide (Lasix) to a patient with renal dysfunction, the nurse will administer the medication no faster than which rate? A. 2 mg/min B. 4 mg/min C. 6 mg/min D. 8 mg/min

B. 4 mg/min Correct Furosemide controlled infusion rate should not exceed at a rate of 4 mg/min in patients with renal failure.

6. 6.ID: 10068753408 While assessing the patient, the nurse notes edema of the hands and feet at +3 from third spacing. The patient is having signs and symptoms of intravascular dehydration, and the total protein laboratory result is 4.6 g/dL. The nurse anticipates the health care provider will prescribe which IV solution for this patient? A. Whole blood B. 5% albumin C. Normal saline D. Lactated Ringer's solution

B. 5% albumin Correct If the total protein level falls below 5.3 g/dL, fluid shifts out of blood vessels into the tissues. When this happens, colloid replacement therapy is required to reverse this process by increasing the colloid oncotic pressure. The three most commonly used are 5% albumin, dextran 40, and hetastarch. They all have a very rapid onset of action as well as a long duration of action.

3. 3.ID: 10068743969 When teaching a patient about beta blockers such as atenolol (Tenormin) and metoprolol (Lopressor), it is important for the nurse to instruct the patient about which drug information? A. Alcohol intake is encouraged for its vasodilating effects. B. Abrupt medication withdrawal may lead to a rebound hypertension. C. Hot baths and showers will help increase the therapeutic effects of this drug. D. These medications may be taken with antacids to minimize gastrointestinal distress.

B. Abrupt medication withdrawal may lead to a rebound hypertension. Correct Abrupt withdrawal of a beta-blocking drug can cause rebound hypertension. These drugs should not be withdrawn abruptly, but should be tapered over 1 to 2 weeks. Antacids should not be taken with beta blockers because they may decrease absorption.

3. 3.ID: 10068762588 While obtaining a patient history, the nurse notes that the patient has been prescribed ethosuximide (Zarontin). The nurse suspects that the patient has which type of seizure? A. Partial B. Absence C. Tonic-clonic D. Generalized

B. Absence Ethosuximide is used to treat uncomplicated absence seizures.

1. 1.ID: 10068756870 The nurse would question a prescription for somatropin (Genotropin) in a patient with which condition? A. Dwarfism B. Acromegaly C. Growth failure D. Hypopituitarism

B. Acromegaly Correct Somatropin is a synthetic form of human growth hormone. Acromegaly is caused by excessive growth hormone, and thus this drug would be contraindicated.

A patient with atrial fibrillation requests a cup of chamomile tea. The nurse denies the patient's request because of the medication the patient is taking. What is the medication? A. Cardiac glycoside B. Anticoagulants C. Beta-adrenergic blocker D. Calcium channel blocker

B. Anticoagulants The nurse denies the patient's request for chamomile tea because chamomile can increase the risk of bleeding in a patient taking anticoagulants. Cardiac glycoside, beta-adrenergic blockers, and calcium channel blockers are affected by herbal products by increasing the chances of heart block or dysrhythmia.

12. 12.ID: 10068746649 When applying nitroglycerin (Nitro-Bid) ointment, the nurse should perform which action? A. Massage and then gently rub the ointment into the skin. B. Apply the ointment to a nonhairy part of the upper torso. C. Apply a thick layer of ointment on the nitroglycerin paper. D. Use the fingers to spread the ointment evenly over a 3-inch area.

B. Apply the ointment to a nonhairy part of the upper torso. Correct Use the proper dosing paper supplied by the drug company to apply a thin layer of ointment on clean, dry, hairless skin of the upper arms or body. Avoid areas below the knees and elbows. Wear gloves to avoid contact with the skin and subsequent absorption. Do notrub the ointment into the skin; cover the area with an occlusive dressing if not provided (e.g., plastic wrap).

2. 2.ID: 10068743963 The nurse knows which drug is an example of a cardioselective beta blocker? A. Sotalol (Betapace) B. Atenolol (Tenormin) C. Propranolol (Inderal) D. Labetalol (Normodyne)

B. Atenolol (Tenormin) Correct At therapeutic dosages, atenolol selectively blocks only the beta1 receptors in the heart, not the beta2 receptors located in the lungs.

4. 4.ID: 10068752044 When performing discharge teaching for a client prescribed oral linezolid (Zyvox) to treat methicillin-resistant Staphylococcus aureus (MRSA), the nurse should emphasize which important information? A. Stop the drug as soon as you feel better. B. Avoid ingestion of foods containing tyramine. C. Report any occurrence of constipation or facial flushing. D. Take the drug with an antacid to avoid gastrointestinal (GI) upset.

B. Avoid ingestion of foods containing tyramine. Correct Hypertension may occur in clients consuming tyramine-containing foods such as aged cheese or wine, soy sauce, smoked meats or fish, and sauerkraut while taking linezolid. Linezolid causes diarrhea, not constipation, and should be taken with food to decrease GI distress. An antacid would interfere with absorption.

14. 14.ID: 10068749231 The nurse will assess a patient receiving gemfibrozil (Lopid) and warfarin (Coumadin) for the increased risk of which adverse effect? A. Clotting B. Bleeding C. Vitamin K toxicity D. Deep vein thrombosis

B. Bleeding Gemfibrozil can bind with vitamin K in the intestinal tract, reducing vitamin K absorption. Because vitamin K is the antidote for warfarin, a lack of vitamin K increases the anticoagulant effect of warfarin and thus the risk of bleeding.

2. 2.ID: 10068753486 Selective serotonin reuptake inhibitors (SSRIs) and tricyclic antidepressants (TCAs) both function by which mechanism? A. Decrease the catecholamine release into the blood B. Block the reuptake of neurotransmitters at nerve endings C. Inhibit an enzyme that stops the action of neurotransmitters D. Stimulate areas of the brain associated with mental alertness

B. Block the reuptake of neurotransmitters at nerve endings Correct The SSRIs block the reuptake of serotonin. The TCAs block the reuptake of norepinephrine and serotonin. The monoamine oxidase inhibitors (MAOIs) inhibit the MAO enzyme that stops the actions of neurotransmitters such as dopamine, serotonin, and norepinephrine. Amphetamines stimulate areas of the brain associated with mental alertness.

1. 1.ID: 10068752054 For a client receiving an intravenous (IV) infusion of gentamicin (Garamycin), the nurse would monitor which laboratory values? A. Hematocrit and hemoglobin B. Blood urea nitrogen (BUN) and creatinine C. Prothrombin time and partial thromboplastin time D. Serum glutamic-oxaloacetic transaminase and alanine transaminase

B. Blood urea nitrogen (BUN) and creatinine Gentamicin has a high potential for nephrotoxicity. Nephrotoxicity typically occurs in 5% to 25% of clients. Thus, the client's renal function test results for BUN and creatinine must be monitored closely throughout therapy.

7. 7.ID: 10068753470 A patient prescribed lorazepam (Ativan) for the treatment of anxiety states, "I feel drowsy all the time, and it's interfering with every aspect of my life." The nurse knows that a better drug therapy option for this patient is which anxiolytic medication? A. Alprazolam (Xanax) B. Buspirone (BuSpar) C. Chlordiazepoxide (Librium) D. Hydroxyzine hydrochloride salt (Vistaril)

B. Buspirone (BuSpar) Correct Buspirone (BuSpar) has the advantage of having fewer adverse effects such as sedation and lack of dependency potential. All of the other options are sedatives as well as anxiolytics.

8. 8.ID: 10068761788 Which drug class is used to treat both hypertension and antidysrhythmias? A. Sodium channel blockers B. Calcium channel blockers C. Direct-acting vasodilators D. Alpha-adrenergic-blocking

B. Calcium channel blockers Correct Calcium channel blockers are effective in treating both hypertension and dysrhythmias secondary to their negative inotropic and chronotropic effects.

4. 4.ID: 10068745783 Before administering eplerenone (Inspra) to a patient, what is the priority nursing action? A. Auscultate heart sounds. B. Check serum electrolytes. C. Assess level of consciousness. D. Obtain serum thiocyanate level.

B. Check serum electrolytes. Correct Eplerenone (Inspra) is contraindicated in patients with an elevated serum potassium level (greater than 5.5 mEq/L) or severe renal impairment. Therefore, it is imperative that the nurse assess the patient's most recent serum electrolytes before administering this medication.

9. 9.ID: 10068753480 Which laboratory test should be monitored closely to assess for a potential life-threatening adverse effect to clozapine (Clozaril)? A. Liver function studies B. Complete blood count C. Immunoglobulin levels D. Glomerular filtration rate

B. Complete blood count Patients taking clozapine must be monitored for the life-threatening adverse effect of agranulocytosis, evidenced by a severe reduction in the number of white blood cells.

6. 6.ID: 10068761786 The patient is prescribed ibutilide (Corvert), a class III antiarrhythmic drug. The nurse understands this drug has been prescribed for which reason? A. Conversion of life-threatening ventricular dysrhythmias B. Conversion of recent-onset atrial fibrillation and flutter C. Treatment of dysrhythmias in patients with acute renal failure D. Treatment of PSVT

B. Conversion of recent-onset atrial fibrillation and flutter Correct Ibutilide is specifically indicated only for treatment of recent-onset atrial fibrillation and flutter.

4. 4.ID: 10068745077 The nurse is providing care to a client prescribed a nonselective adrenergic agonist bronchodilator. Which condition documented in the client's medical history would alert the nurse to question this prescription? A. Thrombocytopenia B. Coronary artery disease C. Mycobacterium tuberculosis D. Chronic obstructive pulmonary disease

B. Coronary artery disease Correct Nonselective adrenergic agonist bronchodilators stimulate beta1 receptors in the heart and beta2 receptors in the lungs. Stimulation of beta1 receptors can increase heart rate and contractility, increasing oxygen demand. This increased oxygen demand may lead to angina or myocardial ischemia in clients with coronary artery disease.

7. 7.ID: 10068745707 After administering oxybutynin (Ditropan) to a patient with spina bifida, the nurse is monitoring the patient for therapeutic effects. Which assessment finding indicates the therapeutic effect of this drug? A. Increase in heart rate B. Decrease in urinary frequency C. Absence of muscle rigidity and tremors D. Sudden urge to have a bowel movement

B. Decrease in urinary frequency Correct Oxybutynin (Ditropan) is a synthetic antimuscarinic drug used for the treatment of overactive bladder. It is also used as an antispasmodic for neurogenic bladder associated with spinal cord injuries and congenital conditions such as spina bifida.

13. 13.ID: 10068746665 A patient receiving IV nitroglycerin at 20 mcg/min complains of dizziness. Nursing assessment reveals a blood pressure of 85/40 mm Hg, heart rate of 110 beats/min, and respiratory rate of 16 breaths/min. What is the nurse's best action? A. Assess the patient's lung sounds. B. Decrease the IV nitroglycerin by 10 mcg/min. C. Increase the IV nitroglycerin by 10 mcg/min. D. Recheck the patient's vital signs in 1 hour.

B. Decrease the IV nitroglycerin by 10 mcg/min. Correct Nitroglycerin, as a vasodilator, causes a decrease in blood pressure. Because it is short-acting, decreasing the infusion rate will allow the blood pressure to rise. The patient should be monitored every 10 minutes while changing the rate of the IV nitroglycerin infusion.

13. 13.ID: 10068745775 When teaching a patient about a new prescription for carvedilol (Coreg), the nurse explains that this medication reduces blood pressure by which action? (Select all that apply.) A. Relaxes muscle tone B. Decreases heart rate C. Peripheral vasodilation D. Increases urine output E. Promotes excretion of sodium

B. Decreases heart rate C. Peripheral vasodilation Carvedilol (Coreg) has the dual antihypertensive effects of reduction in heart rate (beta1 receptor blockade) and vasodilation (alpha1receptor blockade).

8. 8.ID: 10068745781 The nurse is conducting a community education program. When explaining different medication regimens to treat hypertension, it would be accurate to state that African Americans probably respond best to which combination of medications? A. ACE inhibitors and diuretics B. Diuretics and calcium channel blockers C. Diuretics and beta blockers D. ACE inhibitors and beta blockers

B. Diuretics and calcium channel blockers Research has demonstrated that African Americans do not typically respond therapeutically to beta blockers or ACE inhibitors. They respond better to diuretics and calcium channel blockers.

3. 3.ID: 10068749243 Hydroxymethylglutaryl-coenzyme A (HMG-CoA) reductase inhibitors (statins) are generally administered at which time of day? A. Morning B. Evening C. Afternoon D. 12:00 noon

B. Evening The liver produces the majority of cholesterol during the night. Thus, statin drugs, which decrease the cholesterol synthesis, are generally administered in the evening or bedtime so that the peak drug levels coincide with cholesterol production.

2. 2.ID: 10068756816 The health care provider has prescribed dopamine (Intropin) to treat the patient's hypovolemic shock secondary to severe blood loss. For the medication to be effective, the health care provider must also prescribe which treatment? A. Temporary pacing B. Fluid replacement C. Induced hypothermia D. Beta-stimulating drugs

B. Fluid replacement Correct Dopamine increases blood pressure secondary to vasoconstriction, which has a limited effect if there is not enough volume within the circulatory system.

1. 1.ID: 10068762594 The nurse instructs a patient receiving phenytoin (Dilantin) to visit the dentist regularly and perform frequent oral hygiene. What common adverse effect is the nurse educating the patient about for this medication? A. Oral candidiasis B. Gingival hyperplasia C. Increased risk of tooth abscess D. Increased incidence of dental caries

B. Gingival hyperplasia A well-known adverse effect of long-term oral phenytoin therapy is overgrowth of gum tissue, or gingival hyperplasia. This can be minimized by frequent oral hygiene.

12. 12.ID: 10068760542 When caring for a pregnant patient with gestational diabetes, the nurse should question a prescription for which drug? A. Insulin glargine (Lantus) B. Glipizide (Glucotrol) C. Insulin glulisine (Apidra) D. NPH insulin

B. Glipizide (Glucotrol) Oral antidiabetic drugs are classified as pregnancy B or C drugs and are generally not recommended for pregnant patients.

1. 1.ID: 10068758818 For a patient prescribed hydrochlorothiazide (HydroDIURIL), the nurse should closely monitor which laboratory test value? A. Sodium B. Glucose C. Calcium D. Chloride

B. Glucose Thiazide and thiazide-like diuretics are associated with adverse metabolic effects of hyperglycemia; therefore, close monitoring of blood glucose levels are needed. Other metabolic disturbances include hyperlipidemia and hyperuricemia.

11. 11.ID: 10068758812 When assessing a patient taking triamterene (Dyrenium), the nurse would monitor for which possible adverse effect? A. Hypokalemia B. Hyperkalemia C. Hypoglycemia D. Hypernatremia

B. Hyperkalemia Correct Triamterene is a potassium-sparing diuretic, and therefore hyperkalemia is a possible adverse effect.

6. 6.ID: 10068745777 The nurse should question a prescription for a calcium channel blocker in a patient with which condition? A. Dysrhythmia B. Hypotension C. Angina pectoris D. Increased intracranial pressure

B. Hypotension Correct Calcium channel blockers cause smooth muscle vasodilation and thus a drop in blood pressure. They are contraindicated in the presence of hypotension.

In developing a plan of care for a patient receiving morphine sulfate (MS Contin), which nursing diagnosis has the highest priority? A. Acute pain related to metastatic tumor cancer B. Impaired gas exchange related to respiratory depression C. Constipation related to decreased GI motility D. Risk for injury related to CNS adverse effects

B. Impaired gas exchange related to respiratory depression Correct Using Maslow's hierarchy of needs and the ABCs of prioritization, impaired gas exchange is a priority over pain, constipation, and a risk for injury. If a patient cannot oxygenate sufficiently, all of the other problems will not matter because the patient will not live to worry about them.

5. 5.ID: 10068745703 What would be a priority nursing diagnosis for a patient receiving anticholinergic (parasympatholytic) drugs? A. Risk for injury related to excessive central nervous system stimulation B. Impaired gas exchange related to thickened respiratory secretions C. Urinary retention related to loss of bladder tone D. Deficient knowledge related to pharmacologic regimen

B. Impaired gas exchange related to thickened respiratory secretions Correct Although all of these nursing diagnoses are appropriate, the priority is determined remembering the ABCs. Anticholinergic drugs decrease respiratory secretions, which could lead to mucous plugs and resultant impaired gas exchange.

A patient is prescribed an opioid analgesic for chronic pain. Which information should the nurse discuss with the patient to minimize the GI adverse effects? A. Avoid eating foods high in lactobacilli. B. Increase fluid intake and fiber in the diet. C. Take the medication on an empty stomach. D. Take diphenoxylate-atropine (Lomotil) with each dose.

B. Increase fluid intake and fiber in the diet. Opioid analgesics decrease GI intestinal motility (peristalsis), leading to constipation. Increasing fluid and fiber in the diet or use of stool softener or mild laxative can prevent constipation.

3. 3.ID: 10068758190 For a patient taking levothyroxine (Synthroid) and warfarin (Coumadin) concurrently, the nurse would closely monitor for which possible serious adverse effect? A. Acute confusion B. Increased bruising C. Cardiac dysrhythmias D. Orthostatic hypotension

B. Increased bruising Correct Levothyroxine can compete with protein-binding sites of warfarin, allowing more warfarin to be unbound or free, thus increasing effects of warfarin and risk of bleeding. Bleeding commonly presents as bruising.

1. 1.ID: 10068760516 Which is a rapid-acting insulin with an onset of action of less than 15 minutes? A. Insulin glargine (Lantus) B. Insulin aspart (NovoLog) C. Insulin detemir (Levemir) D. Regular insulin (Humulin R)

B. Insulin aspart (NovoLog) Correct Insulin aspart is a rapid-acting insulin. Insulin glargine and insulin detemir are long-acting insulins. Regular insulin is short acting.

2. 2.ID: 10068758802 The nurse would suspect a patient is taking too much levothyroxine (Synthroid) when the patient exhibits which adverse effect? A. Lethargy B. Irritability C. Feeling cold D. Weight gain

B. Irritability Correct Irritability is a symptom of hyperthyroidism and may indicate toxicity of the medication. The other choices are signs of hypothyroidism.

12. 12.ID: 10068745067 Before administering an LTRA medication, the nurse would assess the client for allergies to which substance? (Select all that apply.) A. Latex B. Lactose C. Cellulose D. Povidone E. Chlorhexidine

B. Lactose Correct C. Cellulose Correct D. Povidone Correct Allergies to povidone, lactose, titanium dioxide, or cellulose derivatives are important to note because these are inactive ingredients in LTRAs.

9. 9.ID: 10068752050 A client who is prescribed metronidazole (Flagyl) for a gynecologic infection provides the nurse with a list of medications that are routinely taken. Which medication would lead the nurse to question the prescription for Flagyl? A. Ibuprofen (Advil) B. Lithium (Eskalith) C. Levothyroxine (Synthroid) D. Multivitamin (Thera-Tabs)

B. Lithium (Eskalith) Concomitant use of lithium and metronidazole may result in lithium toxicity. Thus, a client who reports taking lithium should alert the nurse to notify the health care provider because of the potential significant interaction.

A patient who takes ginseng to improve memory takes many prescription drugs as well. Which laboratory parameter should the nurse check before administering medication as a means of assessing the pharmacokinetic effect of ginseng in this patient? A. Mental status B. Liver enzymes C. Urine bilirubin D. Serum creatinine

B. Liver enzymes Ginseng can alter drug metabolism and, as a result, drug distribution and elimination. To evaluate the potential for altered pharmacokinetics of the patient's prescription drugs, the nurse checks the liver enzymes as an indicator of liver function because the liver is the main site of drug metabolism in the body. Herbals must be used cautiously, may be contraindicated in many patients' status and affect laboratory values.

2. 2.ID: 10068753400 When caring for a patient with a serum potassium of 2.8 mEq/L, what is the priority nursing intervention when giving IV replacement therapy? A. Administer potassium as a bolus over 10 minutes. B. Maintain infusion rate at no greater than 20 mEq/hr. C. Apply ice packs to site of IV administration. D. Teach the patient signs and symptoms of hypokalemia.

B. Maintain infusion rate at no greater than 20 mEq/hr. Too rapid an infusion of potassium may cause cardiac arrest. Therefore, IV potassium infusion rates should not exceed 20 mEq/hr.

5. 5.ID: 10068762586 While completing discharge teaching for a patient prescribed an antiepileptic drug, the nurse instructs the patient of which potential complication if the medication is stopped abruptly? A. Confusion and delirium B. Rebound seizure activity C. Orthostatic hypotension D. Acute withdrawal syndrome

B. Rebound seizure activity Abrupt withdrawal of antiepileptic drugs can cause rebound seizure activity.

13. 13.ID: 10068760518 Which information should the nurse include in a teaching plan for patients taking oral hypoglycemic drugs? (Select all that apply.) A. Take your medication only as needed. B. Report symptoms of anorexia and fatigue. C. Explain dietary changes are not necessary. D. Advise to avoid smoking and alcohol consumption. E. Instruct that it is okay to skip breakfast 1 to 2 times per week.

B. Report symptoms of anorexia and fatigue. D. Advise to avoid smoking and alcohol consumption. Oral hypoglycemic drugs must be taken on a daily scheduled basis to maintain euglycemia and prevent long-term complications of diabetes. Skipping meals can cause low blood glucose levels and should be avoided. Patients with type 2 diabetes mellitus are managed with lifestyle changes. All other options are correct.

3. 3.ID: 10068741267 Which antiparkinson drug causes an increase in the levels of dopaminergic stimulation in the central nervous system (CNS) and therefore allows a decreased dose of other medications? A. Tolcapone (Tasmar) B. Selegiline (Eldepryl) C. Diphenhydramine (Benadryl) D. Carbidopa- levodopa (Sinemet)

B. Selegiline (Eldepryl) Selegiline is a selective monoamine oxidase (MAO) B inhibitor that has been shown to cause an increase in the levels of dopaminergic stimulation in the CNS and thus allow the dose of levodopa to be decreased.

9. 9.ID: 10068746643 The nurse is teaching treatment of acute chest pain for a patient prescribed nitroglycerin (Nitrostat) sublingual tablets. Which instructions should the nurse include? A. Take five tablets every 3 minutes for chest pain. B. Sit or lie down before taking medication. C. Chew or swallow the tablet for the quickest effect. D. Keep the tablets locked in a safe place until you need them.

B. Sit or lie down before taking medication. Correct Nitroglycerin is a vasodilator and can cause orthostatic hypotension, resulting in dizziness. It should be kept in a readily accessible location for immediate use should chest pain occur. Three tablets may be taken 5 minutes apart. It should be placed under the tongue and allowed to dissolve.

6. 6.ID: 10068753488 What is another approved and indicated use for the antidepressant bupropion (Zyban)? A. Nocturnal enuresis B. Smoking cessation C. Tourette's syndrome D. Orthostatic hypotension

B. Smoking cessation Zyban is a sustained-release form of bupropion that is useful in helping patients to quit smoking.

9. 9.ID: 10068745759 The nurse is providing education to a patient on the primary uses of cholinergic drugs. Which uses would the nurse include in the teaching? (Select all that apply.) A. Increase heart rate B. Stimulate peristalsis C. Dilate pulmonary airways D. Stimulate bladder emptying E. Decrease intraocular pressure

B. Stimulate peristalsis Correct D. Stimulate bladder emptying Correct E. Decrease intraocular pressure Correct Cholinergic drugs are used primarily for their effects on the GI tract, bladder, and eye. These drugs stimulate the intestine and bladder, which results in increased gastric secretions, GI motility, and urinary frequency. They also stimulate constriction of the pupil, or miosis. This helps decrease intraocular pressure. In addition, cholinergic drugs cause increased salivation and sweating. Cardiovascular effects include reduced heart rate and vasodilation. Pulmonary effects include causing the bronchi of the lungs to constrict and the airways to narrow.

4. 4.ID: 10068752898 A patient receiving an IV infusion of one unit of packed red blood cells suddenly develops shortness of breath, chills, and is feeling hot. What is the nurse's priority action? A. Check the vital signs. B. Stop the blood transfusion. C. Notify the health care provider. D. Maintain a patent IV line with normal saline.

B. Stop the blood transfusion. Correct These are symptoms of a blood transfusion reaction. Thus, the nurse's priority action is to immediately stop the blood transfusion.

7. 7.ID: 10068758886 During antibiotic therapy, the nurse will assess the client for a condition that may occur because of the disruption of normal flora. The nurse knows this as what condition? A. Organ toxicity B. Superinfection C. Hypersensitivity D. Allergic reaction

B. Superinfection Superinfections can occur when antibiotic therapy reduces or completely eliminates the normal bacterial flora of the body, which normally would inhibit the overgrowth of fungi and yeast. When the normal bacterial are flora and are reduced or completely eliminated, these organisms can overgrow and cause infections.

1. 1.ID: 10068745075 The nurse receives laboratory values for a client with a theophylline level of 14 mcg/mL. How does the nurse interpret this theophylline level? A. Toxic B. Therapeutic C. Subtherapeutic D. Life threatening

B. Therapeutic Correct The therapeutic theophylline level is 10 to 20 mcg/mL.

3. 3.ID: 10068753484 A patient diagnosed with depression is being discharged with a prescription for TCAs after no improvement of symptoms on an SSRI. Which instruction should the nurse include about this new medication? A. There are no drug or food contraindications with this medication. B. There is a risk of toxicity when this medication is taken with alcohol. C. Take St. John's wort every day to minimize the adverse effects of the medication. D. This drug does not cause problems with sleep, constipation, or low blood pressure.

B. There is a risk of toxicity when this medication is taken with alcohol. Correct There is an increased risk of toxicity with TCAs when taken with alcohol and a high rate of morbidity.

2. 2.ID: 10068758834 The nurse is providing education to a patient prescribed spironolactone (Aldactone) and furosemide (Lasix). What information does the nurse explain to the patient? A. Using two drugs increases blood osmolality and the glomerular filtration rate. B. This combination promotes diuresis but decreases the risk of low levels of potassium. C. The lowest dose of two different types of diuretics are more effective than a large dose of one type. D. This combination maintains water balance to protect against dehydration and electrolyte imbalance.

B. This combination promotes diuresis but decreases the risk of low levels of potassium. Correct Spironolactone is a potassium-sparing diuretic; furosemide is a potassium-losing diuretic. Giving these together minimizes potassium loss.

2. 2.ID: 10068745755 Cholinergic (parasympathomimetic) drugs are indicated for which situation? A. To treat a postoperative patient who has bradycardia B. To lower intraocular pressure in patients with glaucoma C. To manage patients with excessive salivation and sweating D. For conditions in which the patient has frequent urination

B. To lower intraocular pressure in patients with glaucoma Cholinergic drugs stimulate the pupil to constrict (miosis), thus decreasing intraocular pressure.

5. 5.ID: 10068746661 Which dosage form of nitroglycerin has the longest duration of action? A. Sublingual tablet B. Transdermal patch C. Intravenous (IV) infusion D. Immediate-release tablet

B. Transdermal patch Correct The transdermal patch has an 8- to 12-hour duration of action compared with 3 minutes to 6 hours for the other dosage forms of nitroglycerin.

The nurse should monitor an older adult patient prescribed a benzodiazepine for treatment of insomnia for which potential adverse effect? A. Red rash B. Unsteady gait C. Muscle spasms D. Shortness of breath

B. Unsteady gait Benzodiazepine doses for children and older adults should be small with gradual increases to avoid ataxia (unsteady gait) and excessive sedation. Thus, these patients should be closely monitored for these adverse effects.

9. 9.ID: 10068753440 When teaching a patient regarding the administration of digoxin (Lanoxin), the nurse instructs the patient not to take this medication with which food? A. Bananas B. Wheat bran C. French toast D. Scrambled eggs

B. Wheat bran Encourage patients to avoid using antacids or eating ice cream, milk products, yogurt, cheese (dairy products), or bran for 2 hours before or 2 hours after taking medication to avoid interference with the drugs absorption.

2. 2.ID: 10068758894 Which information should the nurse include in discharge teaching for a client prescribed doxycycline (Vibramycin)? A. "Keep the remainder of the medication in case of recurrence." B. "Take the medication until you have no fever and feel better." C. "Apply sunscreen or wear protective clothing when outdoors." D. "Take the medication with milk to minimize gastrointestinal upset."

C. "Apply sunscreen or wear protective clothing when outdoors." Photosensitivity is a common adverse effect of doxycycline, a tetracycline antibiotic. The client should avoid direct sun exposure and tanning bed use while taking this medication. Exposure to the sun can cause severe burns.

10. 10.ID: 10068746645 The nurse is providing discharge teaching for a patient with a new prescription for nitroglycerin (Nitrostat) sublingual tablets. Which statement by the patient indicates an understanding of the nurse's discharge instructions about this medication? A. "My nitroglycerin tablets are not affected by cold or heat." B. "I can take some aspirin if I get a headache related to nitroglycerin." C. "I will keep my nitroglycerin tablets in their original glass container." D. "I will need to refill my prescription when I feel burning under my tongue."

C. "I will keep my nitroglycerin tablets in their original glass container." Correct The sublingual dosage form of nitroglycerin needs to be kept in its original amber-colored glass container with metal lid to avoid loss of potency from exposure to heat, light, moisture, and cotton filler. It should be replaced every 3 to 6 months in order to maintain potency. Potency of the sublingual nitroglycerin is noted if there is burning or stinging when the medication is placed under the tongue; if the medication does not burn, then the drug has lost its potency, and a new prescription must be obtained. Headaches associated with nitrates last approximately 20 minutes (with sublingual forms) and may be managed with acetaminophen.

7. 7.ID: 10068749245 Which statement by the patient indicates a need for further instruction about colestipol (Colestid) from the nurse? A. "The potential adverse effects of this drug are rash and itching." B. "I might need to take fat-soluble vitamins to supplement my diet." C. "I will mix and stir the powder thoroughly with at least 1 to 2 oz of fluid." D. "I should take this medication 1 hour after or 4 hours before my other medications."

C. "I will mix and stir the powder thoroughly with at least 1 to 2 oz of fluid." Correct Colestipol is available in powder form that must be mixed thoroughly with food or fluids (at least 4 to 6 oz of fluid) before administration to avoid esophageal irritation or obstruction and intestinal obstruction.

8. 8.ID: 10068743955 When performing discharge instructions to a patient prescribed metoprolol (Lopressor), which statement by the nurse is correct? A. "If you become dizzy, do not take your medication for 2 days and then restart on the third day." B. "This medication may make you fatigued; increasing caffeine in your diet may help alleviate this problem." C. "If you take your pulse and it is less than 60 beats/min, hold your medicine and call your health care provider for instructions." D. "Increase your intake of green leafy vegetables to prevent bleeding problems that can be caused by this medication."

C. "If you take your pulse and it is less than 60 beats/min, hold your medicine and call your health care provider for instructions." Beta blockers have a negative chronotropic effect and could cause symptomatic bradycardia or heart block. The health care provider should be consulted before administering to a patient with bradycardia (heart rate

A patient prescribed massage therapy for musculoskeletal pain asks the nurse, "How is rubbing my muscles going to make the pain go away?" What is the nurse's best response? A. "Massaging muscles helps relax the contracted fibers and decrease painful stimuli." B. "Massaging muscles decreases the inflammatory response that initiates the painful stimuli." C. "Massaging muscles activates large sensory nerve fibers that send signals to the spinal cord to close the gate, thus blocking painful stimuli from reaching the brain." D. "Massaging muscles activates small sensory nerve fibers that send signals to the spinal cord to open the gate and allow endorphins to reach the muscles and relieve the pain."

C. "Massaging muscles activates large sensory nerve fibers that send signals to the spinal cord to close the gate, thus blocking painful stimuli from reaching the brain." The gate theory of pain control identifies large sensory nerve fibers that, when stimulated, send signals to the spinal cord to close the gate, blocking pain stimuli from reaching the brain. Therefore, the patient is not having the sensation of pain even if the stimulus is still present.

During patient teaching, the nurse explains the difference between a sedative and hypnotic with which statement? A. "There really is no difference; the terms are used interchangeably." B. "Whereas sedative drugs induce sleep, hypnotic drugs induce a state of hypnosis." C. "Most drugs produce sedation at low doses and sleep, the hypnotic effect, at higher doses." D. "Sedatives are much stronger than hypnotic drugs and should only be used for short periods of time."

C. "Most drugs produce sedation at low doses and sleep, the hypnotic effect, at higher doses." Many drugs have both sedative and hypnotic properties, with the sedative properties evident at low doses and the hypnotic properties demonstrated at larger doses.

11. 11.ID: 10068746671 A patient who is taking nitroglycerin (Nitrostat) sublingual tablets is complaining of flushing and headaches. What is the nurse's best response? A. "Put a cold wet washcloth or use an icepack on your forehead and lie down in a quiet place." B. "Stop taking the nitroglycerin because you are experiencing an allergic reaction to the medication." C. "These are the most common adverse effects of nitroglycerin. They should subside with continued use of nitroglycerin." D. "Immediately notify your health care provider because these symptoms are not related to the sublingual nitroglycerin."

C. "These are the most common adverse effects of nitroglycerin. They should subside with continued use of nitroglycerin." Correct Headache, flushing of the face, dizziness, and fainting are the most common adverse effects of nitroglycerin and the headache generally subsides after the start of therapy.

7. 7.ID: 10068758830 A patient asks the nurse about using potassium supplements while taking spironolactone (Aldactone). What is the nurse's best response? A. "I will call your health care provider and discuss your concern." B. "I would recommend that you take two multivitamins every day." C. "This diuretic is potassium sparing, so there is no need for extra potassium." D. "You will need to take potassium supplements for the medication to be effective."

C. "This diuretic is potassium sparing, so there is no need for extra potassium." Correct Spironolactone is a potassium-sparing diuretic, and thus the patient does not need potassium supplementation. Intake of excess potassium may lead to hyperkalemia.

6. 6.ID: 10068758186 Which patient statement demonstrates understanding of radioactive iodine (I-131) therapy? A. "I will need to take this drug on a daily basis for at least 1 year." B. "This drug will help decrease my cold intolerance and weight gain." C. "This drug will be taken up by the thyroid gland and destroy thyroid tissue." D. "I will isolate myself from my family for 1 week so there is no risk of radiation exposure."

C. "This drug will be taken up by the thyroid gland and destroy thyroid tissue." Radioactive iodine is an antithyroid medication that is administered orally. It concentrates in the thyroid gland, where its radioactivity destroys thyroid tissue.

10. 10.ID: 10068745065 The nurse performs discharge teaching with a client who is prescribed the anticholinergic inhaler ipratropium bromide (Atrovent). Which statement by the client indicates to the nurse that teaching has been successful? A. "I will not drink grapefruit juice while taking this drug." B. "I may gain weight as a result of taking this medication." C. "This inhaler is not to be used alone to treat an acute asthma attack." D. "Nausea and vomiting are common adverse effects of this medication."

C. "This inhaler is not to be used alone to treat an acute asthma attack." Correct Although ipratropium works to prevent bronchoconstriction and thus secondarily leads to bronchodilation, a direct-acting bronchodilator is needed to treat an acute asthma attack.

6. 6.ID: 10068745069 Client teaching regarding the use of leukotriene receptor antagonists (LTRAs) drugs such as zafirlukast (Accolate) would include which statement by the nurse? A. "It will take about 3 or 4 weeks before you notice a therapeutic effect." B. "Take the medication when you are short of breath and begin wheezing." C. "This medication works by preventing the inflammation that causes your asthma attack." D. "Increase fiber and fluid in your diet to prevent the common adverse effect of constipation."

C. "This medication works by preventing the inflammation that causes your asthma attack." Correct LTRAs drugs block the inflammatory response of leukotrienes and thus the trigger for asthma attacks. Response to these drugs is usually noticed within 1 week. They are not used to treat acute asthma attacks. Diarrhea, not constipation, is a common adverse effect of montelukast and zafirlukast

A patient questions the use of epinephrine for repair of a laceration, stating, "I thought that was the drug used in the emergency department for someone who is coding." Which is the nurse's best response? A. "Epinephrine is used with lidocaine to prevent adverse effects." B. "The systemic absorption of lidocaine is maximized by the epinephrine, and the anesthetic effect is reached more quickly." C. "Vasoconstriction caused by epinephrine enhances the duration of action for lidocaine and minimizes bleeding at the laceration site." D. "Epinephrine is metabolized more quickly than lidocaine so that the anesthetic effect wears off more quickly after the laceration is sutured."

C. "Vasoconstriction caused by epinephrine enhances the duration of action for lidocaine and minimizes bleeding at the laceration site." Epinephrine causes localized vasoconstriction, not only allowing for a bloodless field to suture but also delaying absorption of the lidocaine and thus enhancing its numbing effect.

1. 1.ID: 10068745093 The nurse caring for an adult patient with symptomatic bradycardia prepares to administer which recommended dose of intravenous (IV) atropine? A. 2 mg B. 0.3 mg C. 0.5 mg D. 1.25 mg

C. 0.5 mg The recommended dose of atropine to treat symptomatic bradycardia is 0.5 to 1 mg.

2. 2.ID: 10068745713 An adult patient presents to the emergency department with insecticide poisoning. The nurse prepares to administer which dose of IV atropine? A. 0.3 mg B. 0.5 mg C. 2 mg D. 4 mg

C. 2 mg Higher doses of atropine (1 to 3 mg) are needed to treat cholinergic overdoses such as insecticide poisoning.

6. 6.ID: 10068745099 Which would be the most appropriate application time for a patient prescribed a scopolamine (Transderm Scop) transdermal medication patch for motion sickness? A. Just before bedtime B. Every 4 hours as needed C. 4 to 5 hours before travel D. At the first sign of motion sickness

C. 4 to 5 hours before travel Correct For the prevention of motion sickness, scopolamine is available in a transdermal delivery system (Transderm Scop), a patch that can be applied just behind the ear 4 to 5 hours before travel. Each patch has a 72-hour duration of action, thus necessitating a change only every 3 days.

4. 4.ID: 10068760540 Assuming the patient eats breakfast at 8:30 AM, lunch at noon, and dinner at 6:00 AM, he or she is at highest risk of hypoglycemia after an 8:00 AM dose of NPH insulin at what time? A. 10:00 AM B. 2:00 PM C. 5:00 PM D. 8:00 PM

C. 5:00 PM Correct Breakfast eaten at 8:30 AM would cover the onset of NPH insulin, and lunch will cover the 2 PM time frame. However, if the patient does not eat a mid-afternoon snack, the NPH insulin may be peaking just before dinner without sufficient glucose on hand to prevent hypoglycemia.

While admitting a patient for treatment of an acetaminophen (Tylenol) overdose, the nurse prepares to administer which medication to prevent toxicity? A. Naloxone (Narcan) B. Phytonadione (vitamin K) C. Acetylcysteine (Mucomyst) D. Methylprednisolone (Solu-Medrol)

C. Acetylcysteine (Mucomyst) Acetylcysteine is the antidote for acetaminophen overdose. It must be administered as a loading dose followed by subsequent doses every 4 hours for 17 additional doses and started as soon as possible after the acetaminophen ingestion (ideally within 12 hours).

6. 6.ID: 10068746667 To prevent the development of tolerance to nitroglycerin (Nitro-Bid) transdermal patch, the nurse instructs the patient to perform which action? A. Apply a new nitroglycerin patch every other day. B. Use the nitroglycerin patch for acute episodes of angina only. C. Apply the nitroglycerin patch in the morning and remove it at night for 8 hours. D. Switch to sublingual nitroglycerin when the systolic blood pressure is greater than 140 mm Hg.

C. Apply the nitroglycerin patch in the morning and remove it at night for 8 hours. Correct To avoid development of tolerance to transdermal nitroglycerin patches, maintain an 8-hour nitrate-free period each day. A common regimen with transdermal patches is to remove them at night for 8 hours and apply a new patch in the morning.

3. 3.ID: 10068753438 A patient prescribed digoxin (Lanoxin) 0.25 mg and furosemide (Lasix) 40 mg for the treatment of systolic heart failure states, "I am starting to see yellow halos around lights." Which action will the nurse take? A. Perform a visual acuity test on each eye. B. Document the finding and reassess in 1 hour. C. Assess for other symptoms of digoxin toxicity. D. Prepare to administer digoxin immune fab (Digifab).

C. Assess for other symptoms of digoxin toxicity. Yellow-green halos around objects is a symptom of digoxin toxicity. Other signs and symptoms of digoxin toxicity include headache, dizziness, confusion, nausea, and blurred vision. Electrocardiogram findings show heart block, atrial tachycardia with block, or ventricular dysrhythmias.

6. 6.ID: 10068749239 Which statement by the nurse explains to the patient the action of cholestyramine (Questran) to decrease blood lipid levels? A. Inhibits absorption of dietary cholesterol in the small and large intestine. B. Stimulates the biliary system to increase excretion of dietary cholesterol. C. Binds to bile in the intestinal tract, forming an insoluble complex that is excreted in the feces. D. Inhibits lipolysis in adipose tissue and decreases the hepatic synthesis of triglycerides in the liver.

C. Binds to bile in the intestinal tract, forming an insoluble complex that is excreted in the feces. Correct Cholestyramine is an anion exchange resin that binds to bile acids in the small intestine to form an insoluble complex that is excreted in the feces. The liver must then use cholesterol to synthesize more bile.

15. 15.ID: 10068746655 When caring for a patient with angina pectoris, the nurse would question a prescription for a noncardioselective beta blocker in a patient with which preexisting condition? A. Hypertension B. Atrial fibrillation C. Bronchial asthma D. Myocardial infarction

C. Bronchial asthma Correct Noncardioselective beta blockers should be used with caution in patients with bronchial asthma, because any level of blockade of beta2-receptors can promote bronchoconstriction.

4. 4.ID: 10068758874 A client who is allergic to penicillin is at increased risk for an allergy to which drug? A. Erythromycin (E-mycin) B. Gentamicin (Garamycin) C. Cefazolin sodium (Ancef) D. Demeclocycline (Declomycin)

C. Cefazolin sodium (Ancef) Clients who are allergic to penicillins have an increased risk of allergy to other beta-lactam antibiotics. The incidence of cross-reactivity between cephalosporins and penicillins is reported to be between 1% and 4%.

7. 7.ID: 10068758196 When assessing for potential serious adverse effects to propylthiouracil (PTU), the nurse will monitor which laboratory test? A. Kidney function B. Serum electrolytes C. Complete blood count (CBC) D. Brain natriuretic peptide

C. Complete blood count (CBC) Correct With antithyroid medications, the nurse should monitor for possible serious adverse reactions such as agranulocytosis, leukopenia, and thrombocytopenia. An abnormal CBC result would indicate bone marrow dysfunction.

When assessing a patient for adverse effects related to morphine sulfate (MS Contin), which clinical findings is the nurse MOST likely to find? (Select all that apply.) A. Diarrhea B. Weight gain C. Constipation D. Inability to void E. Excessive bruising

C. Constipation D. Inability to void Morphine sulfate causes a decrease in GI motility (delayed gastric emptying and slowed peristalsis). This leads to constipation, not diarrhea. Morphine can also cause urinary retention (inability to void).

9. 9.ID: 10068761790 Calcium channel blockers have which pharmacodynamic effect? A. Positive inotropic B. Positive chronotropic C. Coronary vasodilation D. Shortened refractory period

C. Coronary vasodilation Correct Calcium channel blockers cause coronary vasodilation, a negative inotropic effect, a negative chronotropic effect, and a negative dromotropic effect.

A patient with a diagnosis of pneumonia asks the nurse, "Why am I receiving codeine when I have no pain?" The nurse's response is based on knowledge that codeine also has what effect? A. Expectorant B. Bronchodilation C. Cough suppressant D. Increases sputum production

C. Cough suppressant Codeine provides both analgesic and antitussive (cough suppressant) therapeutic effects.

9. 9.ID: 10068758878 In an effort to prevent superinfections of the GI tract such as Clostridium difficile, the nurse will instruct clients to eat which foods? A. Multigrain wheat bread B. Raw fruits and vegetables C. Cultured dairy products such as yogurt D. Low-fat meats such as chicken and pork

C. Cultured dairy products such as yogurt The natural flora in the GI tract may be killed off by antibiotics, leaving other bacteria such as C. difficile to overgrow. This process may be prevented through consumption of probiotics (e.g., yogurt, buttermilk, kefir).

10. 10.ID: 10068758822 To evaluate the therapeutic effects of mannitol (Osmitrol), the nurse should monitor the patient for which clinical finding? A. Increase in urine osmolality B. Decrease in serum osmolality C. Decrease in intracranial pressure D. Increase in cerebral blood volume

C. Decrease in intracranial pressure Correct Mannitol is an osmotic diuretic that pulls fluid from extravascular spaces into the bloodstream to be excreted in urine. This decreases intracranial pressure and cerebral blood volume, increases excretion of medications, decreases urine osmolality, and increases serum osmolality.

1. 1.ID: 10068753426 The nurse is preparing to administer digoxin (Lanoxin) 0.25mg intravenous push to a patient. Which is an expected patient outcome related to the administration of digoxin? A. Low serum potassium B. Reduction in urine output C. Decrease in the heart rate D. Increase in blood pressure

C. Decrease in the heart rate Correct Digoxin has a negative chronotropic effect (decreased heart rate).

3. 3.ID: 10068745091 The nurse monitors a patient prescribed tolterodine (Detrol) for which therapeutic effect? A. Absence of salivation B. Increase in heart rate C. Decrease in urinary frequency D. Reduction in gastrointestinal (GI) motility

C. Decrease in urinary frequency Correct Tolterodine (Detrol) blocks the muscarinic receptors in the bladder to decrease urinary frequency, urgency, and urge incontinence.

3. 3.ID: 10068756862 The nurse is caring for a patient with diabetes insipidus (DI) who is receiving vasopressin (Pitressin). What therapeutic effect does the nurse expect from this drug? A. Increase in thirst B. Improved skin turgor C. Decrease in urine output D. Normal serum albumin level

C. Decrease in urine output Correct Vasopressin increases the water reabsorption in the kidneys, thus decreasing urine output. It is used to treat DI, which presents with polydipsia, polyuria, and dehydration.

Which entity regulates medicinal herbs? A. FDA B. American Medical Association (AMA) C. Dietary Supplement Health and Education Act of 1994 (DSHEA) D. U.S. Department of Agriculture (USDA)

C. Dietary Supplement Health and Education Act of 1994 (DSHEA) Medicinal herbs are regulated under DSHEA, a piece of legislation created to define dietary supplements and regulations for their sale. The FDA, AMA, and USDA provide no safeguards or regulations surrounding dietary supplements.

9. 9.ID: 10068745779 What is the classification of carvedilol (Coreg)? A. Beta blocker B. Alpha2 blocker C. Dual-action alpha1 and beta receptor blocker D. ACE inhibitor

C. Dual-action alpha1 and beta receptor blocker Carvedilol blocks both the alpha1and beta receptors of the adrenergic nervous system.

14. 14.ID: 10068746647 It is MOST important for the nurse to instruct a patient prescribed nitroglycerin to avoid which substance? A. Antacids B. Grapefruit juice C. Erectile dysfunction drugs D. Potassium-sparing diuretics

C. Erectile dysfunction drugs Correct Concurrent administration of nitrate drugs and erectile dysfunction drugs such as sildenafil citrate (Viagra), tadalafil (Cialis), and vardenafil (Levitra) can cause an additive hypotensive effect.

The nurse teaches a patient prescribed the fentanyl (Duragesic) transdermal delivery system to change the patch at what interval? A. Once a week B. Every 24 hours C. Every 72 hours D. When pain recurs

C. Every 72 hours The fentanyl transdermal delivery system is designed to slowly release analgesic over a 72-hour time frame.

2. 2.ID: 10068756872 Which is a priority nursing diagnosis for a patient receiving desmopressin (DDAVP)? A. Risk for injury B. Acute pain C. Excess fluid volume D. Deficient knowledge regarding medication

C. Excess fluid volume Correct Desmopressin is a form of antidiuretic hormone, which increases sodium and water retention, leading to an alteration in fluid volume. Although the other nursing diagnoses may be appropriate, they are not a priority using Maslow's hierarchy of needs.

5. 5.ID: 10068753414 The nurse is caring for a patient with renal insufficiency and thrombocytopenia. Along with platelet transfusions, the nurse would expect to administer which blood product to increase deficient clotting factors in this patient? A. Albumin B. Whole blood C. Fresh-frozen plasma D. Plasma protein factors

C. Fresh-frozen plasma Correct Fresh-frozen plasma is indicated to increase clotting factors in patients with a known deficiency. Albumin and plasma protein factors do not contain clotting factors. Although whole blood does contain the same ingredients as fresh-frozen plasma, the amount of volume that must be administered to give the patient the necessary clotting factors may be contraindicated in a patient with renal insufficiency.

5. 5.ID: 10068752060 The nurse should assess a client for nephrotoxicity and ototoxicity when administering which antimicrobial? A. Cefazolin (Ancef) B. Clindamycin (Cleocin) C. Gentamicin (Garamycin) D. Erythromycin

C. Gentamicin (Garamycin) Aminoglycoside antibiotics, including gentamicin, have a high risk for nephrotoxicity and ototoxicity.

11. 11.ID: 10068760522 The nurse will instruct the patient to treat hypoglycemia with which drug? A. Acarbose (Precose) B. Propranolol (Inderal) C. Glucagon (GlucaGen) D. Bumetanide (Bumex)

C. Glucagon (GlucaGen) Correct Glucagon stimulates glycogenolysis, raising serum glucose levels.

14. 14.ID: 10068758808 Acetazolamide (Diamox) is used to treat which conditions? (Select all that apply.) A. Dry eye syndrome B. Cardiac dysrhythmias C. High-altitude sickness D. Open-angle glaucoma E. Edema associated with heart failure

C. High-altitude sickness Correct D. Open-angle glaucoma Correct E. Edema associated with heart failure Correct Acetazolamide causes excretion of bicarbonate, which would worsen metabolic acidosis. It is used to treat high-altitude sickness, edema secondary to heart failure, open-angle glaucoma, and rarely as an antiepileptic drug. It is not used for treatment of dry eye syndrome or cardiac dysrhythmias.

10. 10.ID: 10068745787 When administering nitroprusside (Nipride) by continuous intravenous infusion, the nurse monitors for which symptom of drug toxicity? A. Fever B. Wheezing C. Hypotension D. Hyperglycemia

C. Hypotension Correct The main symptom of sodium nitroprusside overdose or toxicity is excessive hypotension.

8. 8.ID: 10068749225 By which action does atorvastatin (Lipitor) decrease lipid levels? A. Stimulating the gallbladder and biliary system to increase excretion of dietary cholesterol B. Binding to bile in the intestinal tract, forming an insoluble complex that is excreted in the feces C. Inhibiting HMG-CoA reductase, the enzyme responsible for the biosynthesis of cholesterol in the liver D. Decreasing the amount of triglycerides produced by the liver and increasing the removal of triglycerides by the liver

C. Inhibiting HMG-CoA reductase, the enzyme responsible for the biosynthesis of cholesterol in the liver Atorvastatin (Lipitor) is an HMG-CoA reductase inhibitor that inhibits HMG-CoA reductase, the enzyme needed to make cholesterol in the liver.

5. 5.ID: 10068749229 What is the mechanism of action of ezetimibe (Zetia)? A. Inhibits the biosynthesis of cholesterol in the liver. B. Decreases the adhesion of cholesterol in the arteries. C. Inhibits absorption of dietary and biliary cholesterol in the small intestine. D. Binds to bile acids in the intestine, inhibiting its reabsorption into the blood.

C. Inhibits absorption of dietary and biliary cholesterol in the small intestine. Correct Ezetimibe selectively inhibits absorption of cholesterol in the small intestine.

4. 4.ID: 10068746651 Calcium channel blockers reduce myocardial oxygen demand by decreasing afterload. How would the nurse explain afterloadto the patient? A. It is the contractility of the heart muscle. B. It is the total volume of blood in the heart. C. It is the force against which the heart must pump. D. It is the pressure within the four chambers of the heart.

C. It is the force against which the heart must pump. Afterload is the force (systemic vascular resistance) against which the heart must exert itself when delivering blood to the body.

9. 9.ID: 10068749251 To assess for a potentially serious adverse effect to HMG-CoA reductase inhibitors, the nurse should monitor which laboratory results? A. Serum electrolytes B. Urine specific gravity C. Liver function studies D. Complete blood count

C. Liver function studies Correct HMG-CoA reductase inhibitors can cause hepatic toxicity; thus, liver function studies are often measured every 6 to 8 weeks for the first 6 months of statin therapy and then every 3 to 6 months, depending on the prescriber and the patient situation.

10. 10.ID: 10068749237 The nurse is providing discharge teaching for a patient about potential serious adverse effects to simvastatin (Zocor). Which symptom may indicate the patient is experiencing a serious adverse effect to this medication? A. Itching B. Headache C. Muscle pain D. Weight loss

C. Muscle pain Correct Unexplained muscle pain and soreness are symptoms of a relatively uncommon but serious adverse effect of rhabdomyolysis associated with statin drugs and must be immediately reported to the health care provider.

4. 4.ID: 10068756868 The nurse admitting a patient with acromegaly anticipates administering which medication? A. Corticotropin (Acthar) B. Desmopressin (DDAVP) C. Octreotide (Sandostatin) D. Somatropin (Genotropin)

C. Octreotide (Sandostatin) Octreotide suppresses growth hormone, the culprit of acromegaly.

5. 5.ID: 10068753432 Which are therapeutic effects of digoxin (Lanoxin)? A. Positive inotropic, positive chronotropic, and negative dromotropic B. Positive inotropic, negative chronotropic, and positive dromotropic C. Positive inotropic, negative chronotropic, and negative dromotropic D. Negative inotropic, negative chronotropic, and negative dromotropic

C. Positive inotropic, negative chronotropic, and negative dromotropic Digoxin increases cardiac contractility (positive inotropic effect), decreases heart rate (negative chronotropic effect), and decreases conductivity (negative dromotropic effect).

6. 6.ID: 10068760534 Which oral hypoglycemic drug has a quick onset and short duration of action, enabling the patient to take the medication 30 minutes before eating and skip the dose if he or she does not eat? A. Acarbose (Precose) B. Metformin (Glucophage) C. Repaglinide (Prandin) D. Pioglitazone (Actos)

C. Repaglinide (Prandin) Repaglinide is known as the "Humalog of oral hypoglycemic drugs." The drug's very fast onset of action allows patients to take the drug with meals and skip a dose when they skip a meal.

10. 10.ID: 10068761792 The nurse is caring for a patient prescribed amiodarone (Cordarone). The nurse knows the MOST serious adverse effect of this medication can occur in which body system? A. Nervous B. Immune C. Respiratory D. Gastrointestinal

C. Respiratory Pulmonary toxicity is the most serious potential adverse effect of amiodarone.

7. 7.ID: 10068743951 What is the priority nursing diagnosis for a patient prescribed metoprolol (Lopressor)? A. Risk for injury related to possible adverse effects of the adrenergic blockers B. Acute confusion related to adverse central nervous system effects of the drug C. Risk for decreased cardiac tissue perfusion related to effects of medication D. Deficient knowledge related to lack of information about the therapeutic regimen

C. Risk for decreased cardiac tissue perfusion related to effects of medication Using the ABCs of prioritization, risk for decreased cardiac tissue perfusion puts the patient at highest risk. Although the other nursing diagnoses are pertinent, they are not the priority.

8. 8.ID: 10068758820 To treat a patient diagnosed with primary hyperaldosteronism, the nurse would expect to administer which diuretic? A. Furosemide (Lasix) B. Acetazolamide (Diamox) C. Spironolactone (Aldactone) D. Hydrochlorothiazide (HydroDIURIL)

C. Spironolactone (Aldactone) Correct Spironolactone is the direct antagonist for aldosterone.

4. 4.ID: 10068753474 A patient who is prescribed duloxetine (Cymbalta) comes to the medical clinic complaining of restlessness, sweating, and tremors. The nurse suspects serotonin syndrome and questions the patient regarding concurrent use of which herbal product or dietary supplement? A. Zinc B. Vitamin E C. St. John's wort D. Glucosamine chondroitin

C. St. John's wort Correct Serotonin syndrome may occur with SSRIs when they are combined with herbal products such as St. John's wort.

1. 1.ID: 10068743957 Nonselective beta blockers may be used to treat hypertension and what other condition? A. Heart failure B. Heart block C. Supraventricular dysrhythmias D. Chronic obstructive pulmonary disease (COPD)

C. Supraventricular dysrhythmias Correct Nonselective beta blockers are used to treat supraventricular dysrhythmias secondary to their negative chronotropic effects (decreasing heart rate). They may exacerbate heart failure, COPD, and heart block secondary to their negative inotropic effect (heart failure), blocking of bronchodilation (COPD), and negative chronotropic effects (heart block).

10. 10.ID: 10068752052 Quinolones are a class of antibiotics known for several significant complications. Which are possible adverse effects with these drugs? (Select all that apply.) A. Ototoxicity B. Nephrotoxicity C. Tendon rupture D. Prolongation of the QT interval E. Abnormal cartilage development in children

C. Tendon rupture D. Prolongation of the QT interval E. Abnormal cartilage development in children Quinolones are not used in prepubescent children because of the risk of cartilage development issues. Quinolones may also cause a cardiac effect that involves prolongation of the QT interval on the electrocardiogram. The use of these medications can result in tendonitis or ruptured tendons in adults. Nephrotoxicity and ototoxicity are not associated with quinolones.

In monitoring a patient for adverse effects related to morphine sulfate (MS Contin), the nurse assesses for stimulation of which area in the central nervous system (CNS)? A. The cough reflex center B. Sympathetic baroreceptors C. The chemoreceptor trigger zone D. Autonomic control over circulation

C. The chemoreceptor trigger zone Morphine sulfate can irritate the gastrointestinal (GI) tract, causing stimulation of the chemoreceptor trigger zone in the brain, which in turn causes nausea and vomiting.

10. 10.ID: 10068758884 Which statement best describes health care-associated infections? A. They develop in more than 15% of hospitalized clients. B. The infection develops in response to various antibiotics. C. The infection was not incubating at the time of admission. D. Clients are admitted to the hospital with an infectious disease.

C. The infection was not incubating at the time of admission. A health care-associated infection is an infection that is acquired during the course of receiving treatment for another condition in a health care facility. The infection is not present or incubating at the time of admission; also known as a nosocomial infection.

7. 7.ID: 10068756830 A patient using Afrin nasal spray complains of worsening cold symptoms and tells the nurse, "I don't understand why this is not working. I am using it almost every 3 hours." The nurse's response is based on knowledge of which drug information? A. Afrin nasal spray may be minimally effective to relieve the nasal congestion. B. The medication needs to be used every 30 minutes for maximum effectiveness. C. The patient is experiencing rebound congestion related to excessive use of the medication. D. Adrenergic decongestants should only be used prophylactically, not to treat acute congestion.

C. The patient is experiencing rebound congestion related to excessive use of the medication. Afrin nasal spray is a sympathomimetic drug with both alpha- and beta-adrenergic effects. The alpha-adrenergic activity is responsible for causing vasoconstriction in the nasal mucosa. However, excessive use of nasal decongestants can lead to greater congestion because of a rebound phenomenon that occurs when use of the product is stopped.

6. 6.ID: 10068741271 The nurse is caring for a patient with Parkinson's disease who has been taking entacapone (Comtan) for the past week to treat an on-off phenomenon. The patient expresses concern over brown-orange urine. What information will the nurse provide to the patient? A. Laboratory tests are needed to determine kidney function. B. Brown discoloration of urine may indicate a urinary tract problem. C. This is a normal adverse effect of entacapone (Comtan) and is not harmful. D. Abdominal ultrasonography will be performed to evaluate for liver damage or failure.

C. This is a normal adverse effect of entacapone (Comtan) and is not harmful. Patients should be instructed that entacapone (Comtan) can turn urine a brownish-orange color, and this is not a harmful adverse effect.

The nurse is preparing to administer an intravenous injection of morphine to a patient. The nurse assesses a respiratory rate of 10 breaths/min. Which action should the nurse perform? A. Administer the next prescribed dose intramuscularly. B. Administer a smaller dose and document in the patient's record. C. Withhold the medication and notify the health care provider. D. Check the pulse oximeter reading and reevaluate respiratory rate in 1 hour.

C. Withhold the medication and notify the health care provider. Respiratory depression is an adverse effect of opioid analgesia. Therefore, because the patient's respiratory rate is below normal, the nurse should withhold the morphine and notify the health care provider.

CH 10 - ANALGESICS

CH 10 - ANALGESICS

CH 11 - ANESTHETICS

CH 11 - ANESTHETICS

CH 12 - CNS DEPRESSANTS; MUSCLE RELAXANTS

CH 12 - CNS DEPRESSANTS; MUSCLE RELAXANTS

CH 14 - ANTIEPILEPTIC

CH 14 - ANTIEPILEPTIC

CH 15 - ANTIPARKINSON

CH 15 - ANTIPARKINSON

CH 16 - PSYCHOTHERAPEUTIC

CH 16 - PSYCHOTHERAPEUTIC

CH 18 - ADRENERGIC

CH 18 - ADRENERGIC

CH 19 - ANTIADRENERGIC

CH 19 - ANTIADRENERGIC

CH 20 - CHOLINERGIC

CH 20 - CHOLINERGIC

CH 21 - ANTICHOLINERGIC

CH 21 - ANTICHOLINERGIC

CH 22 - ANTIHYPERTENSIVE

CH 22 - ANTIHYPERTENSIVE

CH 23 - ANTIANGINAL

CH 23 - ANTIANGINAL

CH 24 - HEART FAILURE

CH 24 - HEART FAILURE

CH 25 - ANTIDYSRHYTHMIC

CH 25 - ANTIDYSRHYTHMIC

CH 27 - ANTILIPMEIC

CH 27 - ANTILIPMEIC

CH 28 - DIURETIC

CH 28 - DIURETIC

CH 29 - FLUID/ELECTROLYTES

CH 29 - FLUID/ELECTROLYTES

CH 30 - PITUITARY

CH 30 - PITUITARY

CH 31 - THYROID

CH 31 - THYROID

CH 32 - ANTIDIABETIC

CH 32 - ANTIDIABETIC

CH 37 - RESPIRATORY

CH 37 - RESPIRATORY

CH 38 - ANTIBIOTICS

CH 38 - ANTIBIOTICS

CH 39 - ANTIBIOTICS

CH 39 - ANTIBIOTICS

CH 7 - OTC,HERBAL

CH 7 - OTC,HERBAL

1. 1.ID: 10068746663 Which instruction should be included in the discharge teaching for a patient with a transdermal nitroglycerin (Nitro-Dur) patch? A. "If you get chest pain, apply a second patch next to the first patch." B. "If you get a headache, remove the patch for 4 hours and then reapply." C. "Make sure to rub a lotion or cream on the skin before putting on a new patch." D. "Apply the patch to a hairless, nonirritated area of the chest, upper arm, back or shoulder."

D. "Apply the patch to a hairless, nonirritated area of the chest, upper arm, back or shoulder." Correct A nitroglycerin patch should be applied to a clean, residue-free, hairless, nonirritated area for the best and most consistent absorption rates. Sites should be rotated to prevent skin irritation, and if headache occurs, the patient should not change the patch removal schedule to avoid these headaches. Sublingual nitroglycerin should be used to treat chest pain.

8. 8.ID: 10068760526 The patient is prescribed 30 units of regular insulin and 70 units of insulin isophane suspension (NPH insulin) subcutaneously every morning. The nurse should provide which instruction to the patient for insulin administration? A. "Inject the needle at a 30-degree angle." B. "Rotate sites at least once or twice a week." C. "Use a 23- to 25-gauge syringe with a 1-inch needle to increase insulin absorption." D. "Draw up the regular insulin into the syringe first, followed by the cloudy NPH insulin."

D. "Draw up the regular insulin into the syringe first, followed by the cloudy NPH insulin." Correct When insulins are mixed, withdraw the regular insulin (clear) first, followed by withdrawing the NPH insulin (cloudy).

1. 1.ID: 10068758194 Which statement by the patient indicates an understanding of discharge instructions given by the nurse about the newly prescribed medication levothyroxine (Synthroid)? A. "I will take a double dose to make up for the missed one." B. "I can expect improvement of my symptoms within 1 week." C. "I will stop the medication immediately if I feel pain or weakness in my muscles." D. "I will take this medication in the morning so it does not affect my sleep at night."

D. "I will take this medication in the morning so it does not affect my sleep at night." Correct Levothyroxine increases basal metabolic rate and thus may cause insomnia. Patients should not double the dose or stop taking the medication abruptly. It may take up to 4 weeks for a therapeutic response to occur.

A patient verbalizes concern to the nurse regarding postoperative nausea and vomiting from the anesthesia. Which is the nurse's best response? A. "Don't worry because you'll be heavily or completely sedated if that occurs." B. "I understand your concern but intermittent vomiting often occurs after surgery." C. "You will need to notify the charge nurse and health care provider if you feel nauseated after surgery." D. "Nausea and vomiting occur less frequently than in the past because of the use of a balanced approach to anesthesia."

D. "Nausea and vomiting occur less frequently than in the past because of the use of a balanced approach to anesthesia." Explain to the patient the advances in anesthesia may help alleviate fear. Adverse effects such as nausea and vomiting are less common because drugs from several different classes rather than a single drug are used to produce anesthesia. This approach is referred to as balanced anesthesia.

7. 7.ID: 10068745773 During discharge teaching, which statement by the nurse would be MOST appropriate for a patient prescribed a transdermal clonidine (Catapres)? A. "Occasional drooling is a common adverse effect of this medication." B. "Prolonged sitting or standing does not cause hypotension symptoms." C. "Your blood pressure should be checked by your health care provider two to three times a week." D. "The patch should be applied to a nonhairy site, and you should not suddenly stop using this drug."

D. "The patch should be applied to a nonhairy site, and you should not suddenly stop using this drug." Correct Transdermal clonidine patches should be applied to nonhairy areas of the skin, and application sites should be rotated. When the patch dosage form is used, it is important to remove the old patch before applying a new one. It must not be discontinued abruptly because it will lead to severe rebound hypertension (sudden high elevation of blood pressure). Prolonged standing can cause venous pooling and hypotension. Dry mouth, not drooling, is a common adverse effect of clonidine.

8. 8.ID: 10068758880 A client prescribed azithromycin (Zithromax) expresses concern regarding GI upset that was experienced when previously prescribed an erythromycin antibiotic. What is the nurse's best response? A. "Take an over-the-counter antiemetic to lessen the nausea." B. "Stop taking the drug if you experience heartburn and diarrhea." C. "I will call the health care provider and request a different antibiotic." D. "This drug is like erythromycin with less gastrointestinal adverse effects."

D. "This drug is like erythromycin with less gastrointestinal adverse effects." Azithromycin is a newer macrolide antibiotic. It has a longer duration of action, as well as fewer and less severe GI adverse effects than erythromycin.

1. 1.ID: 10068741283 A patient with Parkinson's disease who has been positively responding to carbidopa-levodopa (Sinemet) asks the nurse, "Why have I suddenly developed a relapse of my symptoms." Which explanation by the nurse is appropriate? A. "You have developed drug tolerance to your current medication, and a higher dose will need to be prescribed." B. "This is very common and it is temporary. Continue to take the medicine, and you will feel better in a few weeks." C. "You have developed medication toxicity. Thus, the drug dose will be reduced to avoid potentially toxic drug blood levels." D. "This is called the "on-off phenomenon." Your health care provider can change your medication regimen to help manage this effect."

D. "This is called the "on-off phenomenon." Your health care provider can change your medication regimen to help manage this effect." Patients who have been taking levodopa for a period of time may experience periods of symptom return. Changing to Sinemet controlled release or adding another medication can help reduce the on-off phenomenon.

1. 1.ID: 10068745793 The nurse is assessing the patient's knowledge regarding drug therapy prescribed for the treatment of hypertension. Which statement by the patient indicates the need for further teaching by the nurse? A. "I will wear a medical alert bracelet." B. "I will move slowly from a sitting to standing position." C. "Blood pressure drugs can cause changes in sexual functioning." D. "When my blood pressure is over 140/90, I will take my medication."

D. "When my blood pressure is over 140/90, I will take my medication." Correct Patients must adhere to prescribed antihypertensive regimen to prevent end-organ damage. Many patients do not adhere to this regimen because hypertension itself does not cause symptoms, but the medication may produce unwanted adverse effects. Patient teaching is essential.

10. 10.ID: 10068760538 The nurse is providing education to a patient for the prescription glipizide (Glucotrol). The nurse explains this medication is more effective when administered at which time? A. At bedtime B. In the morning C. 15 minutes postprandial D. 30 minutes before a meal

D. 30 minutes before a meal Correct Glipizide works best if given 30 minutes before meals. This allows the timing of the insulin secretion induced by the glipizide to correspond to the elevation in the blood glucose level induced by the meal.

2. 2.ID: 10068749227 To decrease the skin flushing adverse effect reaction of niacin (nicotinic acid), which action should the nurse take? A. Administer niacin with a liquid antacid. B. Give niacin with all other morning medications. C. Apply cold compresses to the head and neck. D. Administer aspirin 30 minutes before each dose.

D. Administer aspirin 30 minutes before each dose. To help minimize the adverse effect flushing of the skin, the patient should take a small dose of aspirin or nonsteroidal antiinflammatory drugs 30 minutes before taking niacin, but only as prescribed or recommended by the health care provider.

7. 7.ID: 10068745763 The health care provider prescribes donepezil (Aricept) for a patient. The patient states to the nurse, "I have no idea why I take this medication." The nurse knows the administration of donepezil (Aricept) is MOST commonly associated with what condition? A. Bladder retention B. Urinary retention C. Parkinson's disease D. Alzheimer's disease

D. Alzheimer's disease Correct Donepezil (Aricept) is a cholinesterase inhibitor drug that works centrally in the brain to increase levels of acetylcholine by inhibiting acetylcholinesterase. It is used in the treatment of mild to moderate Alzheimer's disease.

2. 2.ID: 10068752058 The nurse will question the use of a fluoroquinolone antibiotic in a client already prescribed which medication? A. Furosemide (Lasix) B. Omeprazole (Prilosec) C. Metoprolol (Lopressor) D. Amiodarone (Cordarone)

D. Amiodarone (Cordarone) Correct Dangerous cardiac dysrhythmias are more likely to occur when quinolones are taken by clients receiving class Ia and class III antidysrhythmic drugs such as disopyramide and amiodarone. For this reason, such drug combinations should be avoided.

While taking a patient's history before surgery for a cardiac problem, the patient tells the nurse, "I am addicted to cocaine." The nurse notifies the anesthesiologist of this finding because use of cocaine can cause which effect when a patient is under anesthesia? A. Blood-clotting problems B. Complications during recovery C. Immediate withdrawal symptoms D. Anesthetic-induced complications

D. Anesthetic-induced complications A patient with a history of substance abuse is at higher risk of anesthetic-related complications and will require closer monitoring by the anesthesia team.

5. 5.ID: 10068758826 The nurse would question the use of mannitol (Osmitrol) for which patient condition? A. Increased intraocular pressure B. Oliguria from acute renal failure C. Cerebral edema from head trauma D. Anuria related to end-stage kidney disease

D. Anuria related to end-stage kidney disease Correct Mannitol does not influence urine production; it only increases existing urine output. It is not metabolized but excreted unchanged in the urine by the kidneys. Thus, if no urine is produced (anuria), mannitol is not excreted, which increases blood volume. Excess blood volume may cause the undesirable adverse effect of pulmonary edema.

2. 2.ID: 10068753434 The patient's serum digoxin level is 0.4 ng/mL. How does the nurse interpret this laboratory value result for digoxin? A. Normal therapeutic level B. A toxic serum blood level C. Above the therapeutic level D. Below the therapeutic level

D. Below the therapeutic level Correct Therapeutic serum digoxin levels are 0.5 to 2 ng/mL.

11. 11.ID: 10068745795 A patient who is prescribed tamsulosin (Flomax) does not have a history of hypertension. The nurse knows this medication is also used for what condition? A. Migraine headache B. Pulmonary emboli C. Subarachnoid hemorrhage D. Benign prostatic hyperplasia (BPH)

D. Benign prostatic hyperplasia (BPH) Alpha1 blockers have beneficial in the treatment of BPH. The blocking of alpha1-adrenergic receptors decreases the urine outflow obstruction related to BPH by preventing smooth muscle contractions in the bladder neck and urethra.

13. 13.ID: 10068749235 The nurse would question a prescription for colesevelam (Welchol) in a patient with which condition? A. Glaucoma B. Renal disease C. Hepatic disease D. Bowel obstruction

D. Bowel obstruction Correct Colesevelam (Welchol) is contraindicated in patients with a history of bowel obstruction.

2. 2.ID: 10068741277 When teaching a patient about carbidopa-levodopa (Sinemet), what information should the nurse include about this medication? A. Giving both drugs together minimizes adverse effects and decreases the risk of kidney or liver disease. B. Carbidopa crosses the blood-brain barrier to increase the metabolism of levodopa to dopamine in the brain. C. Carbidopa increases levodopa's conversion in the periphery, enhancing the amount of dopamine available to the brain. D. Carbidopa decreases levodopa's conversion in the periphery, increasing the levodopa available to cross the blood-brain barrier

D. Carbidopa decreases levodopa's conversion in the periphery, increasing the levodopa available to cross the blood-brain barrier. Adding carbidopa to levodopa inhibits the breakdown of levodopa in the periphery, increasing the amount available to cross the blood-brain barrier and decreasing the extrapyramidal adverse effects caused by dopamine in the periphery, resulting in fewer unwanted adverse effects.

A patient is admitted to the emergency department after taking an overdose of a barbiturate 15 minutes before arrival. The nurse can anticipate that which drug will be prescribed? A. Ipecac syrup B. Naloxone (Narcan) C. Flumazenil (Romazicon) D. Charcoal (Activated charcoal)

D. Charcoal (Activated charcoal) There is no antidote for barbiturates. The use of activated charcoal binds to the drug in the stomach and gastrointestinal tract, preventing absorption.

4. 4.ID: 10068745701 The nurse monitors a patient prescribed dicyclomine (Bentyl) for which therapeutic effect? A. Decrease in sweating B. Dilation of the pupils C. Reduction in urinary frequency D. Decrease in GI motility

D. Decrease in GI motility Correct Dicyclomine is a synthetic antispasmodic cholinergic blocker used to decrease GI motility in patients with functional GI disorders such as irritable bowel syndrome.

6. 6.ID: 10068756874 A patient has developed DI after a head injury. Which medication should the nurse anticipate to be prescribed for the management of DI? A. Corticotrophin (Acthar) B. Octreotide (Sandostatin) C. Somatropin (Genotropin) D. Desmopressin (DDAVP)

D. Desmopressin (DDAVP) Vasopressin (Pitressin) and desmopressin (DDAVP) are used to prevent or control polydipsia (excessive thirst), polyuria, and dehydration in patients with DI caused by a deficiency of endogenous antidiuretic hormone.

7. 7.ID: 10068753422 The nurse reviews an adult patient's laboratory values and notes a digoxin level of 11 ng/mL and a serum potassium level of 6.2 mEq/L. The nurse would notify the health care provider and anticipate which medication will be prescribed to administer? A. Sodium polystyrene sulfonate (Kayexalate) B. Atropine C. Epinephrine (Adrenalin) D. Digoxin immune Fab

D. Digoxin immune Fab Correct Digoxin immune Fab is indicated for severe digoxin toxicity in patients with the following clinical findings: hyperkalemia (serum potassium level higher than 5 mEq/L) with digoxin toxicity; life-threatening digoxin overdose (more than 10 mg digoxin in adults; more than 4 mg digoxin in children); and life-threatening cardiac dysrhythmias, sustained ventricular tachycardia or fibrillation, and severe sinus bradycardia or heart block unresponsive to atropine treatment or cardiac pacing.

2. 2.ID: 10068745771 The nurse understands a patient who is treated for hypertension may be switched to an angiotensin receptor blocker (ARB) because of which angiotensin-converting enzyme (ACE) inhibitor adverse effect? A. Fatigue B. Hypokalemia C. Orthostatic hypotension D. Dry, nonproductive cough

D. Dry, nonproductive cough ACE inhibitors block the breakdown of bradykinins and may cause a dry, nonproductive cough. ARBs do not block this breakdown, thus minimizing this adverse effect. ACE inhibitors and ARBs are equally effective for the treatment of hypertension, but ARBs do not cause cough.

A patient is admitted to the hospital with a diagnosis of benzodiazepine overdose. The nurse anticipates the health care provider will prescribe which antidote for benzodiazepine overdose? A. Naloxone (Narcan) B. Naltrexone (ReVia) C. Nalmefene (Revex) D. Flumazenil (Romazicon)

D. Flumazenil (Romazicon) Flumazenil is the antidote for benzodiazepine overdose. The other options are only effective against opioid agonist effects.

7. 7.ID: 10068762592 When administering intravenous (IV) phenytoin (Dilantin), which action should the nurse perform? A. Monitor blood pressure and pulse oximetry. B. Obtain an infusion pump for the medication. C. Administer through peripheral IV sites only. D. Flush the line with normal saline before and after administration to prevent precipitation.

D. Flush the line with normal saline before and after administration to prevent precipitation. Phenytoin is very irritating to veins and incompatible with all fluids except normal saline. Flushing with normal saline before and after minimizes precipitation. You do not need an infusion pump when administering via IV push, and administration via central lines is preferred.

3. 3.ID: 10068753410 During the IV administration of a hypertonic saline solution to treat a patient with severe hyponatremia, the nurse monitors for which signs and symptoms of hypernatremia? A. Vomiting and diarrhea B. Lethargy and hypotension C. Mental confusion and seizures D. Flushed skin and increased thirst

D. Flushed skin and increased thirst Correct Hypertonic saline solutions administered to raise sodium levels may cause hypernatremia. Thus, very close monitoring for signs and symptoms of hypernatremia is needed. Flushed skin and increased thirst are signs and symptoms of hypernatremia. The other choices are signs and symptoms of hyponatremia.

4. 4.ID: 10068758828 Why does the health care provider prescribe furosemide (Lasix) 40 mg twice a day by mouth for a patient with a history of renal insufficiency? A. Furosemide is effective in treating patients with hypoaldosteronism. B. Furosemide helps the kidney with reabsorption of sodium and water. C. Furosemide has an antagonist effect to prevent respiratory alkalosis. D. Furosemide is effective in treating patients with impaired kidney function.

D. Furosemide is effective in treating patients with impaired kidney function. Furosemide effectiveness continues in impaired renal function. When creatinine clearance decreases below 25 L/min (normal is 125 L/min), the loop diuretics can still work.

11. 11.ID: 10068749241 The nurse would question a prescription for simvastatin (Zocor) in a patient with which condition? A. Diabetes B. Leukemia C. Heart failure D. Hepatic disease

D. Hepatic disease Correct Simvastatin (Zocor) can cause an increase in liver enzymes and thus should not be used in patients with preexisting liver disease.

1. 1.ID: 10068753406 The nurse is caring for a patient who has peripheral edema and is hypotensive after receiving 5000 mL of normal saline intravenous (IV). The nurse anticipates administering which IV solution to correct the fluid imbalance? A. D5W B. Lactated Ringer's solution C. 3% saline solution D. Hetastarch (Hespan)

D. Hetastarch (Hespan) Correct The patient needs to increase intravascular fluid volume. Hetastarch will enable this because it is a colloid that will increase osmotic pull from the extravascular spaces to the intravascular space. A 3% saline solution is also hypertonic, but its use is not preferred secondary to risk of hypernatremia.

1. 1.ID: 10068745767 Cholinergic (parasympathomimetic) drugs have which therapeutic effect? A. Pupil dilation B. Urinary retention C. Blood vessel vasoconstriction D. Increased gastrointestinal (GI) motility

D. Increased gastrointestinal (GI) motility Cholinergic effects mimic the parasympathetic nervous system (rest and digest) as opposed to the sympathetic nervous system (fight or flight). Increasing GI motility helps the body digest. Urinary retention, mydriasis (pupil dilation), and blood vessel vasoconstriction are sympathetic nervous system responses.

1. 1.ID: 10068756818 Epinephrine, as an adrenergic (sympathomimetic) drug, produces which therapeutic effect? A. Urinary retention B. Bronchial constriction C. Decreased intestinal motility D. Increased heart rate and contractility

D. Increased heart rate and contractility Epinephrine causes sympathomimetic actions, including increased heart rate and contractility. The other effects listed are parasympathomimetic in nature.

5. 5.ID: 10068745765 The nurse administering donepezil (Aricept) to a patient with Alzheimer's disease understands this drug has which expected therapeutic action? A. Relief of anxiety and restless behavior of the patient B. Helps control associated urinary and fecal incontinence C. Blocks the effect of norepinephrine at the presynaptic cell membrane D. Increases levels of acetylcholine in the brain by blocking its breakdown

D. Increases levels of acetylcholine in the brain by blocking its breakdown Donepezil is used to treat Alzheimer's disease, a disorder of decreased acetylcholine levels in the brain. Donepezil is an indirect-acting anticholinesterase drug.

5. 5.ID: 10068745085 A nurse is providing education to a client taking two different bronchodilator medications. The nurse identifies which characteristic as the advantage of salmeterol (Serevent) over other beta2 agonists such as albuterol (Proventil)? A. Quicker peak action B. Shorter onset of action C. Extended time of action D. Longer duration of action

D. Longer duration of action Correct Salmeterol has a longer duration of action, requiring the client to use it only twice a day instead of three or four times a day with albuterol.

The health care provider prescribes dantrolene (Dantrium) to a patient immediately after surgery. What condition does the nurse expect the patient has experienced? A. Delirium tremens B. Respiratory arrest C. Tonic-clonic seizure D. Malignant hyperthermia

D. Malignant hyperthermia Dantrolene is a direct acting skeletal muscle relaxant and is the drug of choice to treat malignant hyperthermia, a possible complication of generalized anesthesia.

4. 4.ID: 10068762584 While teaching a patient newly diagnosed with a seizure disorder, what does the nurse state as the goal of pharmacologic therapy of this medication? A. Reduce seizure occurrence to one per week. B. Maximize drug dosages to control seizure activity. C. Eradicate all seizure activity and discontinue prescribed medication after the patient is seizure free for 3 months. D. Maximally reducing seizure activity while minimizing the adverse effects of medication therapy.

D. Maximally reducing seizure activity while minimizing the adverse effects of medication therapy. Anticonvulsant medications are used to prevent the convulsive seizures typically associated with epilepsy. These medications have many adverse effects. The goal of antiepileptic drug therapy is to control or prevent seizures while maintaining quality of life with minimal adverse effects.

5. 5.ID: 10068760532 The nurse is preparing a patient for a computed tomography scan using iodine contrast media. Which medication should the nurse question if prescribed one day before the scheduled procedure? A. Acarbose (Precose) B. Pioglitazone (Actos) C. Repaglinide (Prandin) D. Metformin (Glucophage)

D. Metformin (Glucophage) Correct The concurrent use of metformin with iodinated (iodine-containing) radiologic contrast media has been associated with both acute renal failure and lactic acidosis. Therefore, metformin should be discontinued the day of the test and for at least 48 hours after the patient undergoes any radiologic study that requires the use of such contrast media.

The nurse is caring for a patient with opioid addiction. The nurse anticipates that the patient will be prescribed which medication? A. Naloxone (Narcan) B. Meperidine (Demerol) C. Morphine (MS Contin) D. Methadone (Dolophine)

D. Methadone (Dolophine) Methadone is a synthetic opioid analgesic with gentler withdrawal symptoms and is the drug of choice for detoxification treatment.

5. 5.ID: 10068756828 A hypertensive crisis may occur if adrenergic (sympathomimetic) drugs are given along with which of the following drug classes? A. Beta blockers B. Alpha1 blockers C. Direct renin inhibitors D. Monoamine oxidase inhibitors (MAOIs)

D. Monoamine oxidase inhibitors (MAOIs) Adrenergic drugs combined with MAOIs may cause a possibly life-threatening hypertensive crisis. All of the other drugs listed are used to treat hypertension.

A patient needs to switch analgesic drugs secondary to an adverse reaction to the current treatment regimen. The patient is concerned that the new prescription will not provide optimal pain control. The nurse's response is based on knowledge that doses of analgesics are determined using an equianalgesic table with which drug prototype? A. Meperidine B. Fentanyl C. Codeine D. Morphine

D. Morphine An equianalgesic table is a conversion chart for commonly used opioids. It identifies oral and parenteral dosages that provide comparable analgesia. The equianalgesic table identifies dosages of various narcotics that are equal to 10 mg of morphine. It is important to use when changing to a new opioid or different route. Morphine is the drug prototype for all opioid drugs.

10. 10.ID: 10068753476 The nurse is monitoring a patient taking an antipsychotic medication for extrapyramidal symptoms. Which clinical finding indicates an adverse effect of this drug? A. Dry mouth and constipation B. Blood pressure of 80/50 mm Hg C. Presence of myoglobin in the blood D. Muscle cramps of the head and neck

D. Muscle cramps of the head and neck Dystonia, or sudden and painful muscle spasms, is the only extrapyramidal adverse effect listed. The other adverse effects also occur but are not extrapyramidal effects.

12. 12.ID: 10068758832 When teaching a patient about symptoms of hypokalemia, the nurse will instruct the patient to notify the health care provider if which symptom occurs? A. Diaphoresis B. Constipation C. Blurred vision D. Muscle weakness

D. Muscle weakness Correct Muscle weakness is a common symptom of hypokalemia. The other answers are incorrect.

6. 6.ID: 10068743953 The nurse understands beta blockers produce which cardiovascular effects? A. Positive inotropic, positive chronotropic, and positive dromotropic B. Negative inotropic, positive chronotropic, and positive dromotropic C. Positive inotropic, negative chronotropic, and negative dromotropic D. Negative inotropic, negative chronotropic, and negative dromotropic

D. Negative inotropic, negative chronotropic, and negative dromotropic Beta blockers exert a sympatholytic effect, blocking the effects of sympathetic nervous system stimulation, causing a decrease in heart rate (negative chronotropic), conductivity (negative dromotropic), and contractility (negative inotropic).

5. 5.ID: 10068745791 To prevent cerebral artery spasms after a subarachnoid hemorrhage, the nurse anticipates administering which calcium channel blocker? A. Verapamil (Calan) B. Diltiazem (Cardizem) C. Amlodipine (Norvasc) D. Nimodipine (Nimotop)

D. Nimodipine (Nimotop) Nimodipine crosses the blood-brain barrier and has a greater effect on the cerebral arteries than on other arteries in the body; thus, it is indicated for the treatment of cerebral artery spasm after subarachnoid hemorrhage.

4. 4.ID: 10068743959 Which is the pharmacological action of propranolol (Inderal)? A. Beta1-adrenergic antagonist B. Beta2-adrenergic antagonist C. Selective alpha-adrenergic antagonist D. Nonselective beta-adrenergic antagonist

D. Nonselective beta-adrenergic antagonist Correct Propranolol is nonselective; it blocks both beta1 and beta2 receptors at therapeutic doses.

4. 4.ID: 10068762506 The nurse understands adenosine (Adenocard) is used to treat which condition? A. Atrial flutter B. Atrial fibrillation C. Second-degree atrioventricular block D. Paroxysmal supraventricular tachycardia (PSVT)

D. Paroxysmal supraventricular tachycardia (PSVT) Correct The only therapeutic indication of use for adenosine is the treatment of PSVT.

3. 3.ID: 10068758876 Which adverse effect can result if tetracycline is administered to children younger than 8 years of age? A. Drug-induced neurotoxicity B. Delayed growth development C. Gastrointestinal (GI) and rectal bleeding D. Permanent discoloration of the teeth

D. Permanent discoloration of the teeth Tetracycline is contraindicated in children younger than 8 years of age because it can cause permanent discoloration of the adult teeth and tooth enamel, which are still forming in the child.

6. 6.ID: 10068756834 The nurse assesses the patient's IV site, and it has infiltrated during the infusion of dopamine (Intropin). The nurse will prepare which medication to treat this infiltration? A. Naloxone (Narcan) B. Lidocaine (Xylocaine) C. Nitroprusside (Nipride) D. Phentolamine (Regitine)

D. Phentolamine (Regitine) Phentolamine is an alpha blocker that causes vasodilation, thus counteracting the vasoconstrictive effects of the infiltrated dopamine. The vasodilation will increase blood flow to the site and decrease the risk of tissue necrosis.

5. 5.ID: 10068743967 During assessment of a patient diagnosed with pheochromocytoma, the nurse auscultates a blood pressure of 210/110 mm Hg. The nurse would expect to administer which medication? A. Verapamil (Calan) B. Nadolol (Corgard) C. Dobutamine (Dobutrex) D. Phentolamine (Regitine)

D. Phentolamine (Regitine) Correct Phentolamine is a potent alpha-blocking drug specifically effective for treatment of hypertension associated with pheochromocytoma.

3. 3.ID: 10068745789 Which medication should the nurse question if prescribed together with ACE inhibitors? A. Furosemide (Lasix) B. Morphine C. Docusate sodium (Colace) D. Potassium chloride (K-Dur)

D. Potassium chloride (K-Dur) Correct ACE inhibitors block the conversion of angiotensin I to angiotensin II, thus also blocking the stimulus for aldosterone production. Aldosterone is responsible for potassium excretion; thus, a decrease in aldosterone production can result in an increase in serum potassium.

11. 11.ID: 10068758872 A client with a known heart condition is prescribed an antibiotic before a dental procedure. What type of antibiotic therapy is this considered? A. Empiric B. Definitive C. Supportive D. Prophylactic

D. Prophylactic Correct Prophylactic antibiotic therapy is used to prevent infections in individuals who are at high risk of development of an infection during or after a procedure. The antibiotics are given before the procedure for prophylactic treatment.

3. 3.ID: 10068752046 Which is a complication of vancomycin IV infusions? A. Angioedema B. Neurotoxicity C. Cardiomyopathy D. Red man syndrome

D. Red man syndrome Correct When infused too rapidly, clients receiving vancomycin may develop hypotension accompanied by flushing or itching of the head, face, neck, and upper trunk area. This phenomenon is called red man syndrome.

3. 3.ID: 10068760530 Which insulin can be administered by continuous intravenous (IV) infusion? A. Insulin glargine (Lantus) B. Insulin aspart (Novolog) C. Insulin detemir (Levemir) D. Regular insulin (Humulin R)

D. Regular insulin (Humulin R) Regular insulin is the only insulin used for IV therapy.

When assessing for the MOST serious adverse effect to an opioid analgesic, what does the nurse monitor for in this patient? A. Heart rate B. Mental status C. Blood pressure D. Respiratory rate

D. Respiratory rate The most serious adverse effect of opioid analgesics is respiratory depression.

Which nursing diagnosis is appropriate for a patient who has just been administered a sedative-hypnotic drug? A. Risk for infection B. Deficient knowledge C. Imbalanced nutrition D. Risk for injury and falls

D. Risk for injury and falls Sedative-hypnotic drugs cause central nervous system depression, putting the patient at risk for injury and falls.

11. 11.ID: 10068753490 What atypical antipsychotic medication should the nurse anticipate the health care provider prescribing for treatment of refractory schizophrenia? A. Phenelzine (Nardil) B. Prazodone (Desyrel) C. Amoxapine (Asendin) D. Risperidone (Risperdal)

D. Risperidone (Risperdal) Risperidone is effective for schizophrenia, including negative symptoms. The other medications listed are antidepressants.

5. 5.ID: 10068758192 The nurse is teaching a patient taking an antithyroid medication to avoid food items high in iodine. Which food item should the nurse instruct the patient to avoid? A. Milk B. Eggs C. Chicken D. Seafood

D. Seafood Seafood contains high amounts of iodine. The other choices do not.

3. 3.ID: 10068745073 Which herbal product, when taken with theophylline, can decrease theophylline's serum drug levels? A. Garlic B. Echinacea C. Peppermint oil D. St. John's wort

D. St. John's wort St. John's wort has been shown to enhance the rate of theophylline metabolism, thus decreasing serum levels.

A male patient who abuses alcohol tells the nurse that he is treating himself with kava. Which instruction should the nurse include in patient teaching? A. Use another herb besides kava. B. Use either the kava or the alcohol. C. Avoid using the kava with alcohol. D. Stop taking the kava now.

D. Stop taking the kava now. The nurse should instruct the patient to stop taking kava now because kava can enhance the effects of alcohol, a depressant, possibly resulting in respiratory arrest. This is a nursing priority because it involves an immediate, potentially life-threatening issue. The nurse would avoid suggesting the use of a substance that the patient abuses and so in this case would avoid a recommendation to use the kava or the alcohol.

Which postoperative nursing action will help the patient avoid serious complications from general anesthesia? A. Monitor input and output closely. B. Provide a quiet, calm environment. C. Place in a semi-Fowler's position. D. Teach to turn, cough, and deep breathe.

D. Teach to turn, cough, and deep breathe. Frequent turning, coughing, and deep breathing can help prevent postoperative atelectasis, a sequela of generalized anesthesia and mechanical ventilation.

9. 9.ID: 10068741275 A patient with Parkinson's disease discusses with the nurse experiencing insomnia recently. The patient asks the nurse, "Can I take my old prescription of benzodiazepine that is prescribed to treat my insomnia?" What does the nurse know about the use of benzodiazepines in patients taking levodopa (Larodopa)? A. Use of benzodiazepines increases the therapeutic effect of the levodopa and may result in an increase in the symptoms of Parkinson's disease. B. Use of benzodiazepines increases the therapeutic effect of the levodopa and may result in a decrease in the symptoms of Parkinson's disease. C. Use of benzodiazepines decreases the therapeutic effect of the levodopa and may result in a decrease in the symptoms of Parkinson's disease. D. Use of benzodiazepines decreases the therapeutic effect of the levodopa and may result in an increase in the symptoms of Parkinson's disease.

D. Use of benzodiazepines decreases the therapeutic effect of the levodopa and may result in an increase in the symptoms of Parkinson's disease. Benzodiazepines interact with levodopa to cause reduced levodopa effects and an increase in the symptoms of Parkinson's disease.

1. 1.ID: 10068761794 A patient is receiving lidocaine (Xylocaine) by continuous intravenous (IV) infusion. The nurse understands this medication is prescribed for what condition? A. Atrial fibrillation B. Sinus bradycardia C. First-degree heart block D. Ventricular dysrhythmias

D. Ventricular dysrhythmias Correct Lidocaine is a sodium channel blocker drug used specifically to treat ventricular dysrhythmias.

7. 7.ID: 10068741273 A patient who has been taking antiparkinson medications for years begins to have increased symptoms on a constant basis. In documenting these symptoms, what term will the nurse use? A. Chorea B. Dystonia C. On-off phenomenon D. Wearing-off phenomenon

D. Wearing-off phenomenon The wearing-off phenomenon occurs when anti-Parkinson's disease medications begin to lose their effectiveness, despite maximal dosing, as the disease progresses. The symptoms return and continue on a constant basis despite treatment and medications.

A patient receiving narcotic analgesics for chronic pain can minimize the GI side effects by: A. taking Lomotil with each dose. B. eating foods high in lactobacilli. C. taking the medication on an empty stomach. D. increasing fluid and fiber in the diet.

D. increasing fluid and fiber in the diet. Narcotic analgesics decrease intestinal motility, leading to constipation. Increasing fluid and fiber in the diet can prevent constipation.

PHARM FINAL STUDY GUIDE CH 7 - OTC,HERBAL CH 10 - ANALGESICS CH 11 - ANESTHETICS CH 12 - CNS DEPRESSANTS; MUSCLE RELAXANTS CH 14 - ANTIEPILEPTIC CH 15 - ANTIPARKINSON CH 16 - PSYCHOTHERAPEUTIC CH 18 - ADRENERGIC CH 19 - ANTIADRENERGIC CH 20 - CHOLINERGIC CH 21 - ANTICHOLINERGIC CH 22 - ANTIHYPERTENSIVE CH 23 - ANTIANGINAL CH 24 - HEART FAILURE CH 25 - ANTIDYSRHYTHMIC CH 27 - ANTILIPMEIC CH 28 - DIURETIC CH 29 - FLUID/ELECTROLYTES CH 30 - PITUITARY CH 31 - THYROID CH 32 - ANTIDIABETIC CH 37 - RESPIRATORY CH 38 - ANTIBIOTICS CH 39 - ANTIBIOTICS CODE BLUE

PHARM FINAL STUDY GUIDE CH 7 - OTC,HERBAL CH 10 - ANALGESICS CH 11 - ANESTHETICS CH 12 - CNS DEPRESSANTS; MUSCLE RELAXANTS CH 14 - ANTIEPILEPTIC CH 15 - ANTIPARKINSON CH 16 - PSYCHOTHERAPEUTIC CH 18 - ADRENERGIC CH 19 - ANTIADRENERGIC CH 20 - CHOLINERGIC CH 21 - ANTICHOLINERGIC CH 22 - ANTIHYPERTENSIVE CH 23 - ANTIANGINAL CH 24 - HEART FAILURE CH 25 - ANTIDYSRHYTHMIC CH 27 - ANTILIPMEIC CH 28 - DIURETIC CH 29 - FLUID/ELECTROLYTES CH 30 - PITUITARY CH 31 - THYROID CH 32 - ANTIDIABETIC CH 37 - RESPIRATORY CH 38 - ANTIBIOTICS CH 39 - ANTIBIOTICS CODE BLUE

6 The nurse is teaching a patient about the inhaler Advair (salmeterol/fluticasone). Which statements by the patient indicate a correct understanding of this medication? Select all that apply. a "I will rinse my mouth with water after each dose." b "I need to use this inhaler whenever I feel short of breath, but not less than 4 hours between doses." c "This medication is taken twice a day, every 12 hours." d "I can take this inhaler if I get short of breath while exercising." e "I will call my doctor if I notice white patches inside my mouth."

a "I will rinse my mouth with water after each dose." c "This medication is taken twice a day, every 12 hours." e "I will call my doctor if I notice white patches inside my mouth."

6 An elderly patient has been discharged following treatment for a mild case of heart failure. He will be taking a loop diuretic. Which instruction(s) from the nurse are appropriate? (Select all that apply.) a "Take the diuretic at the same time each morning." b "Take the diuretic only if you notice swelling in your feet." c "Be sure to stand up slowly because the medicine may make you feel dizzy if you stand up quickly." d "Drink at least 8 glasses of water each day." e "Here is a list of foods that are high in potassium—you need to avoid these." f "Please call your doctor immediately if you notice muscle weakness or increased dizziness."

a "Take the diuretic at the same time each morning." c "Be sure to stand up slowly because the medicine may make you feel dizzy if you stand up quickly." f "Please call your doctor immediately if you notice muscle weakness or increased dizziness."

5 A patient with type 2 diabetes has a new prescription for repaglinide (Prandin). After 1 week, she calls the office to ask what to do, because she keeps missing meals. "I work right through lunch sometimes, and I'm not sure whether I need to take it. What do I need to do?" What is the nurse's best response? a "You need to try not to skip meals, but if that happens, you will need to skip that dose of Prandin." b "We will probably need to change your prescription to insulin injections because you can't eat meals on a regular basis." c "Go ahead and take the pill when you first remember that you missed it." d "Take both pills with the next meal, and try to eat a little extra to make up for what you missed at lunchtime."

a "You need to try not to skip meals, but if that happens, you will need to skip that dose of Prandin."

2 The surgical nurse is reviewing operative cases scheduled for the day. Which of these patients is more prone to complications from general anesthesia? a A 79-year-old woman who is about to have her gallbladder removed b A 49-year-old male athlete who quit heavy smoking 12 years ago c A 30-year-old woman who is in perfect health but has never had anesthesia d A 50-year-old woman scheduled for outpatient laser surgery for vision correction

a A 79-year-old woman who is about to have her gallbladder removed

1 A patient who has a history of asthma is experiencing an acute episode of shortness of breath and needs to take a medication for immediate relief. The nurse will choose which medication that is appropriate for this situation? a A beta agonist, such as albuterol b An leukotriene receptor antagonist, such as montelukast c A corticosteroid, such as fluticasone d An anticholinergic, such as ipratropium

a A beta agonist, such as albuterol

6 The nurse is reviewing drug interactions with a male patient who has a prescription for isosorbide dinitrate (Isordil) as treatment for angina symptoms. Which substances listed below could potentially result in a drug interaction? (Select all that apply.) a A glass of wine b Thyroid replacement hormone c tadalafil (Cialis), an erectile dysfunction drug d metformin (Glucophage), an antidiabetic drug e carvedilol (Coreg), a beta blocker

a A glass of wine c tadalafil (Cialis), an erectile dysfunction drug e carvedilol (Coreg), a beta blocker

7 A patient is taking metformin for new-onset type 2 diabetes mellitus. When reviewing potential adverse effects, the nurse will include information about: (Select all that apply.) a Abdominal bloating b Nausea c Diarrhea d Headache e Weight gain f Metallic taste

a Abdominal bloating b Nausea c Diarrhea f Metallic taste

4 A patient is recovering from general anesthesia. What is the nurse's main concern during the immediate postoperative period? a Airway b Pupillary reflexes c Return of sensations d Level of consciousness

a Airway

2 When giving antihypertensive drugs, the nurse will consider giving the first dose at bedtime for which class of drugs? a Alpha blockers such as doxazosin (Cardura) b Diuretics such as furosemide (Lasix) c ACE inhibitors such as captopril (Capoten) d Vasodilators such as hydralazine (Apresoline)

a Alpha blockers such as doxazosin (Cardura)

6 The nurse is reviewing the orders for wound care, which include use of an antiseptic. Which statements best describe the use of antiseptics? (Select all that apply.) a Antiseptics are appropriate for use on living tissue. b Antiseptics work by sterilizing the surface of the wound. c Antiseptics are applied to nonliving objects to kill microorganisms. d The patient's allergies must be assessed before using the antiseptic. e Antiseptics are used to inhibit the growth of microorganisms on the wound surface.

a Antiseptics are appropriate for use on living tissue. d The patient's allergies must be assessed before using the antiseptic. e Antiseptics are used to inhibit the growth of microorganisms on the wound surface.

3 Which response would the nurse expect to find in a patient with a phenytoin (Dilantin) level of 35 mcg/mL? a Ataxia b Hypertension c Seizures d No unusual response; this level is therapeutic.

a Ataxia

1 The nurse is providing education about cholinergic-blocking drug therapy to an elderly patient. Which is an important point to emphasize for this patient? a Avoid exposure to high temperatures. b Limit liquid intake to avoid fluid overload. c Begin an exercise program to avoid adverse effects. d Stop the medication if excessive mouth dryness occurs.

a Avoid exposure to high temperatures.

4 The nurse will monitor the patient who is taking a muscle relaxant for which adverse effect? a CNS depression b Hypertension c Peripheral edema d Blurred vision

a CNS depression

3 A newly admitted patient reports a penicillin allergy. The prescriber has ordered a second-generation cephalosporin as part of the therapy. Which nursing action is appropriate? a Call the prescriber to clarify the order because of the patient's allergy. b Give the medication, and monitor for adverse effects. c Ask the pharmacy to change the order to a first-generation cephalosporin. d Administer the drug with a nonsteroidal antiinflammatory drug to reduce adverse effects.

a Call the prescriber to clarify the order because of the patient's allergy.

2 When administering niacin, the nurse needs to monitor for which adverse effect? a Cutaneous flushing b Muscle pain c Headache d Constipation

a Cutaneous flushing

3 When monitoring a patient who is receiving an intravenous infusion of nesiritide (Natrecor), the nurse will look for which adverse effect? a Dysrhythmia b Proteinuria c Hyperglycemia d Hypertension

a Dysrhythmia

1 Which action by the nurse is most appropriate for the patient receiving an infusion of packed red blood cells? a Flush the IV line with normal saline before the blood is added to the infusion. b Flush the IV line with dextrose before the blood is added to the infusion. c Check the patient's vital signs once the infusion is completed. d Anticipate that flushed skin and fever are expected reactions to a blood transfusion.

a Flush the IV line with normal saline before the blood is added to the infusion.

1 Which is the most appropriate timing regarding the nurse's administration of a rapid-acting insulin to a hospitalized patient? a Give it 15 minutes before the patient begins a meal. b Give it ½ hour before a meal. c Give it 1 hour after a meal. d The timing of the insulin injection does not matter with insulin lispro.

a Give it 15 minutes before the patient begins a meal.

1 The nurse caring for a patient who is receiving beta1 agonist drug therapy needs to be aware that these drugs cause which effect? a Increased cardiac contractility b Decreased heart rate c Bronchoconstriction d Increased GI tract motility

a Increased cardiac contractility

6 The nurse is monitoring a patient who is receiving an infusion of a beta-adrenergic agonist. Which adverse effects may occur with this infusion? (Select all that apply.) a Mild tremors b Bradycardia c Tachycardia d Palpitations e Drowsiness f Nervousness

a Mild tremors c Tachycardia d Palpitations f Nervousness

6 The nurse is administering an antiepileptic drug and will follow which guidelines? (Select all that apply.) a Monitor the patient for drowsiness. b Medications may be stopped if seizure activity disappears. c Give the medication at the same time every day. d Give the medication on an empty stomach. e Notify the prescriber if the patient is unable to take the medication.

a Monitor the patient for drowsiness. c Give the medication at the same time every day. e Notify the prescriber if the patient is unable to take the medication.

3 A patient with bone pain caused by metastatic cancer will be receiving transdermal fentanyl patches. The patient asks the nurse what benefits these patches have. The nurse's best response includes which of these features? a More constant drug levels for analgesia b Less constipation and minimal dry mouth c Less drowsiness than with oral opioids d Lower dependency potential and no major adverse effects

a More constant drug levels for analgesia

6 The nurse is assessing a patient who has begun therapy with amantadine (Symmetrel) for Parkinson's disease. The nurse will look for which possible adverse effects? (Select all that apply.) a Nausea b Palpitations c Dizziness d Insomnia e Edema

a Nausea c Dizziness d Insomnia

2 A patient is taking entacapone (Comtan) as part of the therapy for Parkinson's disease. Which intervention by the nurse is appropriate at this time? a Notify the patient that this drug causes discoloration of the urine. b Limit the patient's intake of tyramine-containing foods. c Monitor results of renal studies because this drug can seriously affect renal function. d Force fluids to prevent dehydration.

a Notify the patient that this drug causes discoloration of the urine.

6 A patient is taking an alpha blocker as treatment for benign prostatic hyperplasia. The nurse will monitor for which potential drug effects? (Select all that apply.) a Orthostatic hypotension b Increased blood pressure c Increased urine flow d Headaches e Bradycardia

a Orthostatic hypotension c Increased urine flow d Headaches

6 When teaching a patient who has a new prescription for thyroid hormone, the nurse will instruct the patient to notify the physician if which adverse effects are noted? (Select all that apply.) a Palpitations b Weight gain c Angina d Fatigue e Cold intolerance

a Palpitations c Angina

5 A patient tells the nurse that he has been using an herbal supplement that contains kava for several years to help him to relax in the evening. However, the nurse notes that he has a yellow tinge to his skin and sclera, and is concerned about liver toxicity. The nurse advises the patient to stop taking the kava and to see his health care provider for an examination. What else, if anything, should the nurse do at this time? a Report this incident to MedWatch. b Notify the state's pharmaceutical board. c Contact the supplement manufacturer. d No other action is needed.

a Report this incident to MedWatch.

5 The nurse is giving medications to a patient. Which drug or drug class, when administered with lithium, increases the risk for lithium toxicity? a Thiazides b levofloxacin c calcium citrate d Beta blockers

a Thiazides

4 A patient took an accidental overdose of a cholinergic drug while at home. He comes to the emergency department with severe abdominal cramping and bloody diarrhea. The nurse expects that which drug will be used to treat this patient? a atropine (generic) b physostigmine (Antilirium) c bethanechol (Urecholine) d phentolamine (Regitine)

a atropine (generic)

2 When monitoring laboratory test results for patients receiving loop and thiazide diuretics, the nurse knows to look for a decreased serum levels of potassium. b increased serum levels of calcium. c decreased serum levels of glucose. d increased serum levels of sodium.

a decreased serum levels of potassium.

4 A patient is taking pregabalin (Lyrica) but does not have a history of seizures. The nurse recognizes that this drug is also indicated for a postherpetic neuralgia. b viral infections. c Parkinson's disease. d depression.

a postherpetic neuralgia.

5 A patient with Parkinson's disease will start taking entacapone (Comtan) along with the carbidopa-levodopa (Sinemet) he has been taking for a few years. The nurse recognizes that the advantage of taking entacapone is that a the entacapone can reduce on-off effects. b the levodopa may be stopped in a few days. c there is less GI upset with entacapone. d it does not cause the cheese effect.

a the entacapone can reduce on-off effects.

11. 11.ID: 10068753424 For a patient receiving a positive inotropic drug, which nursing assessments should be performed? (Select all that apply.) a. Check apical pulse. b. Obtain daily weights. c. Auscultate lung sounds. d. Monitor serum electrolytes. e. Review red blood cell count.

a. Check apical pulse. Correct b. Obtain daily weights. Correct c. Auscultate lung sounds. Correct d. Monitor serum electrolytes. Correct Lung sounds and daily weights are appropriate assessments related to the treatment of heart failure with inotropic drugs. The apical pulse and serum electrolytes are important assessments related to potential adverse reactions (bradycardia, toxicity with hypokalemia).

9. 9.ID: 10068756882 When teaching a patient the adverse effects of desmopressin (DDAVP), the nurse will instruct the patient to monitor for which potential adverse effects? (Select all that apply.) a. Headache b. Weight gain c. Hypotension d. Nasal irritation e. Hyperglycemia

a. Headache Correct b. Weight gain Correct d. Nasal irritation Correct Desmopressin works to decrease urine output; thus, the patient could retain fluid and gain weight. Other common adverse effects include increased blood pressure, fever, headache, abdominal cramps, and nausea. Desmopressin does not affect serum glucose levels. Because it is administered intranasally, it can be irritating; thus, nostrils should be rotated.

9. 9.ID: 10068758804 The nurse is reviewing the adverse effects of antithyroid medications for a patient prescribed propylthiouracil (PTU). What potential serious adverse effects should the nurse discuss with the patient during discharge teaching? (Select all that apply.) a. Joint pain b. Liver toxicity c. Kidney damage d. Increased urination e. Bone marrow toxicity

a. Joint pain b. Liver toxicity e. Bone marrow toxicity The most damaging or serious adverse effects of the antithyroid medications are liver and bone marrow toxicity. Myalgias and arthralgias (joint pain) may also occur with PTU.

6 A patient has been given a prescription for transdermal scopolamine patches (Transderm-Scōp) for motion sickness for use during a vacation cruise. The nurse will include which instructions? (Select all that apply.) a "Apply the patch as soon as you board the ship." b "Apply the patch 3 to 4 hours before boarding the ship." c "The patch needs to be placed on a nonhairy area on your upper chest or upper arm." d "The patch needs to be placed on a nonhairy area just behind your ear." e "Change the patch every 3 days." f "Rotate the application sites."

b "Apply the patch 3 to 4 hours before boarding the ship." d "The patch needs to be placed on a nonhairy area just behind your ear." e "Change the patch every 3 days." f "Rotate the application sites."

4 Which statement needs to be included when the nurse provides patient education for a patient with heart failure who is taking daily doses of spironolactone (Aldactone)? a "Be sure to eat foods that are high in potassium." b "Avoid foods that are high in potassium." c "Avoid grapefruit juice while taking this medication." d "A low-fiber diet will help prevent adverse effects of this medication."

b "Avoid foods that are high in potassium."

6 When the nurse is teaching a patient about taking an antidysrhythmic drug, which statements by the nurse are correct? (Select all that apply.) a "Take the medication with an antacid if stomach upset occurs." b "Do not chew sustained-release capsules." c "If weight gain of 5 pounds within 1 week occurs, notify your physician at the next office visit." d "If you experience severe adverse effects, stop the drug and notify your physician." e "You may take the medication with food if stomach upset occurs."

b "Do not chew sustained-release capsules." e "You may take the medication with food if stomach upset occurs."

5 A 68-year-old man has been taking the nitrate isosorbide dinitrate (Isordil) for 2 years for angina. He recently has been experiencing erectile dysfunction and wants a prescription for sildenafil (Viagra). Which response would the nurse most likely hear from the prescriber? a "He will have to be switched to isosorbide mononitrate if he wants to take sildenafil." b "Taking sildenafil with the nitrate may result in severe hypotension, so a contraindication exists." c "I'll write a prescription, but if he uses it, he needs to stop taking the isosorbide for one dose." d "These drugs are compatible with each other, and so I'll write a prescription."

b "Taking sildenafil with the nitrate may result in severe hypotension, so a contraindication exists."

3 A 56-year-old man started antihypertensive drug therapy 3 months earlier and is in the office for a follow-up visit. While the nurse is taking his blood pressure, he informs the nurse that he has had some problems with sexual intercourse. Which would be the most appropriate response by the nurse? a "Not to worry. Eventually, tolerance will develop." b "The physician can work with you on changing the dose and/or drugs." c "Sexual dysfunction happens with this therapy, and you will learn to accept it." d "This is an unusual occurrence, but it is important to stay on your medications."

b "The physician can work with you on changing the dose and/or drugs."

4 When making a home visit to a patient who was recently discharged from the hospital, the nurse notes that she has a small pack over her chest and that the pack has a strong odor. She also is drinking herbal tea. When asked about the pack and the tea, the patient says, "Oh, my grandmother never used medicines from the doctor. She told me that this plaster and tea were all I would need to fix things." Which response by the nurse is most appropriate? a "You really should listen to what the doctor told you if you want to get better." b "What's in the plaster and the tea? When do you usually use them?" c "These herbal remedies rarely work, but if you want to use them, then it is your choice." d "It's fine if you want to use this home remedy, as long as you use it with your prescription medicines."

b "What's in the plaster and the tea? When do you usually use them?"

5 The nurse is teaching a patient who has a new prescription for the antithyroid drug propylthiouracil (PTU). Which statement by the nurse is correct? a "There are no food restrictions while on this drug." b "You need to avoid foods high in iodine, such as iodized salt, seafood, and soy products." c "This drug is given to raise the thyroid hormone levels in your blood." d "Take this drug in the morning on an empty stomach."

b "You need to avoid foods high in iodine, such as iodized salt, seafood, and soy products."

5 A patient is currently taking a statin. The nurse considers that the patient may have a higher risk of developing rhabdomyolysis when also taking which product? a NSAIDs b A fibric acid derivative c Orange juice d Fat soluble vitamins

b A fibric acid derivative

5 A patient has a new prescription for an ACE inhibitor. During a review of the patient's list of current medications, which would cause concern for a possible interaction with this new prescription? (Select all that apply.) a A benzodiazepine taken as needed for allergies b A potassium supplement taken daily c An oral anticoagulant taken daily d An opioid used for occasional severe pain e An NSAID taken as needed for headaches

b A potassium supplement taken daily e An NSAID taken as needed for headaches

1 A nurse administering niacin would implement which action to help to reduce adverse effects? a Give the medication with grapefruit juice. b Administer a small dose of aspirin or an NSAID 30 minutes before the niacin dose. c Administer the medication on an empty stomach. d Have the patient increase dietary fiber intake.

b Administer a small dose of aspirin or an NSAID 30 minutes before the niacin dose.

1 The nurse is preparing to give medications. Which is the most appropriate nursing action for intravenous (IV) phenytoin (Dilantin)? a Give IV doses via rapid IV push. b Administer in normal saline solutions. c Administer in dextrose solutions. d Ensure continuous infusion of the drug.

b Administer in normal saline solutions.

6 When checking a patient's fingerstick blood glucose level, the nurse obtains a reading of 42 mg/dL. The patient is awake but states he feels a bit "cloudy-headed." After double-checking the patient's glucose level and getting the same reading, which action by the nurse is most appropriate? a Administer two packets of table sugar. b Administer oral glucose in the form of a semisolid gel. c Administer 50% dextrose IV push. d Administer the morning dose of lispro insulin.

b Administer oral glucose in the form of a semisolid gel.

6 The nurse is administering an IV solution that contains potassium chloride to a patient in the critical care unit who has a severely decreased serum potassium level. Which action(s) by the nurse are appropriate? (Select all that apply.) a Administer the potassium by slow IV bolus. b Administer the potassium at a rate no faster than 20 mEq/hr. c Monitor the patient's cardiac rhythm with a heart monitor. d Use an infusion pump for the administration of IV potassium chloride. e Administer the potassium IV push.

b Administer the potassium at a rate no faster than 20 mEq/hr. c Monitor the patient's cardiac rhythm with a heart monitor. d Use an infusion pump for the administration of IV potassium chloride.

6 The nurse is administering cholestyramine (Questran), a bile acid sequestrant. Which nursing intervention(s) is appropriate? (Select all that apply.) a Administering the drug on an empty stomach b Administering the drug with meals c Instructing the patient to follow a low-fiber diet while taking this drug d Instructing the patient to take a fiber supplement while taking this drug e Increasing fluid intake f Not administering this drug at the same time as other drugs

b Administering the drug with meals d Instructing the patient to take a fiber supplement while taking this drug e Increasing fluid intake f Not administering this drug at the same time as other drugs

1 A patient has been admitted to the emergency department because of an overdose of an oral benzodiazepine. He is very drowsy but still responsive. The nurse will prepare for which immediate intervention? a Hemodialysis to remove the medication b Administration of flumazenil c Administration of naloxone d Intubation and mechanical ventilation

b Administration of flumazenil

3 The nurse is preparing to administer a medication for sleep. Which intervention applies to the administration of a nonbenzodiazepine, such as zaleplon (Sonata)? a These drugs need to be taken about 1 hour before bedtime. b Because of their rapid onset, these drugs need to be taken just before bedtime. c The patient needs to be cautioned about the high incidence of morning drowsiness that may occur after taking these drugs. d These drugs are less likely to interact with alcohol.

b Because of their rapid onset, these drugs need to be taken just before bedtime.

4 A patient is taking a beta blocker as part of the treatment plan for heart failure. The nurse knows that the purpose of the beta blocker for this patient is to a increase urine output. b prevent stimulation of the heart by catecholamines. c increase the contractility of the heart muscle. d cause peripheral vasodilation.

b Bradycardia

4 Intravenous morphine is prescribed for a patient who has had surgery. The nurse informs the patient that which common adverse effects can occur with this medication? (Select all that apply.) a Diarrhea b Constipation c Pruritus d Urinary frequency e Nausea

b Constipation c Pruritus e Nausea

5 When a drug is characterized as having a negative chronotropic effect, the nurse knows to expect which effect? a Reduced blood pressure b Decreased heart rate c Decreased ectopic beats d Increased force of cardiac contractions

b Decreased heart rate

3 When assessing for adverse effects of cholinergic-blocking drug therapy, the nurse would expect to find that the patient complains of which drug effect? a Diaphoresis b Dry mouth c Diarrhea d Urinary frequency

b Dry mouth

1 The nurse is administering antihypertensive drugs to older adult patients. The nurse knows that which adverse effect is of most concern for these patients? a Dry mouth b Hypotension c Restlessness d Constipation

b Hypotension

1 The nurse is reviewing the use of bethanechol (Urecholine) in a patient who is experiencing postoperative urinary retention. Which statement best describes the mechanism of action of bethanechol? a It causes decreased bladder tone and motility. b It causes increased bladder tone and motility. c It increases the sensation of a full bladder. d It causes the sphincters in the bladder to become tighter.

b It causes increased bladder tone and motility.

4 A patient will be taking selegiline (Eldepryl), 10 mg daily, in addition to dopamine replacement therapy for Parkinson's disease. The nurse will implement which precautions regarding selegiline? a Teach the patient to avoid foods containing tyramine. b Monitor for dizziness. c Inform the patient that this drug may cause urine discoloration. d Monitor for tachycardia and palpitations.

b Monitor for dizziness.

7 An opioid analgesic is prescribed for a patient. The nurse checks the patient's medical history knowing this medication is contraindicated in which disorder? a Renal insufficiency b Severe asthma c Liver disease d Diabetes mellitus

b Severe asthma

3 A patient has been receiving an aminophylline (xanthine derivative) infusion for 24 hours. The nurse will assess for which adverse effect when assessing the patient during the infusion? a CNS depression b Sinus tachycardia c Increased appetite d Temporary urinary retention

b Sinus tachycardia

7 During a patient's recovery from a lengthy surgery, the nurse monitors for signs of malignant hyperthermia. In addition to a rapid rise in body temperature, which assessment findings would indicate the possible presence of this condition? (Select all that apply.) a Respiratory depression b Tachypnea c Tachycardia d Seizure activity e Muscle rigidity

b Tachypnea c Tachycardia e Muscle rigidity

6 The nurse is reviewing the drug history of a patient, and during the interview the patient asks, "Why are some drugs over-the-counter and others are not?" The nurse keeps in mind that criteria for over-the-counter status include: (Select all that apply.) a The condition must be diagnosed by a health care provider. b The benefits of correct usage of the drug outweigh the risks. c The drug has limited interaction with other drugs. d The drug is easy to use. e The drug company sells OTC drugs at lower prices.

b The benefits of correct usage of the drug outweigh the risks. c The drug has limited interaction with other drugs. d The drug is easy to use.

6 The nurse is teaching a patient about treatment with an SSRI antidepressant. Which teaching considerations are appropriate? (Select all that apply.) a The patient should be told which foods contain tyramine and instructed to avoid these foods. b The patient should be instructed to use caution when standing up from a sitting position. c The patient should not take any products that contain the herbal product St. John's wort. d This medication should not be stopped abruptly. e Drug levels may become toxic if dehydration occurs. f The patient should be told to check with the prescriber before taking any over-the-counter medications.

b The patient should be instructed to use caution when standing up from a sitting position. c The patient should not take any products that contain the herbal product St. John's wort. d This medication should not be stopped abruptly. f The patient should be told to check with the prescriber before taking any over-the-counter medications.

1 A patient with a rapid, irregular heart rhythm is being treated in the emergency department with adenosine. During administration of this drug, the nurse will be prepared to monitor the patient for which effect? a Nausea and vomiting b Transitory asystole c Muscle tetany d Hypertension

b Transitory asystole

1 When a patient has experienced extravasation of a peripheral infusion of dopamine, the nurse will inject the alpha blocker phentolamine (Regitine) into the area of extravasation and expect which effect? a Vasoconstriction b Vasodilation c Analgesia d Hypotension

b Vasodilation

4 The nurse is caring for a patient who has been taking clozapine (Clozaril) for 2 months. Which laboratory test(s) should be performed regularly while the patient is taking this medication? a Platelet count b WBC count c Liver function studies d Renal function studies

b WBC count

6 Which drugs have an action similar to that of the naturally occurring hormone ADH? (Select all that apply.) a cosyntropin (Cortrosyn) b desmopressin (DDAVP) c somatropin (Humatrope) d vasopressin (Pitressin) e octreotide (Sandostatin)

b desmopressin (DDAVP) d vasopressin (Pitressin)

1 When teaching the patient about the signs and symptoms of cardiac glycoside toxicity, the nurse should alert the patient to watch for a visual changes such as photophobia. b flickering lights or halos around lights. c dizziness when standing up. d increased urine output.

b flickering lights or halos around lights.

6 When giving intravenous cholinergic drugs, the nurse must watch for symptoms of a cholinergic crisis, such as: (Select all that apply.) a peripheral tingling. b hypotension. c dry mouth. d syncope. e dyspnea. f tinnitus.

b hypotension. d syncope. e dyspnea.

3 The nurse is giving a dose of bethanechol (Urecholine) to a postoperative patient. The nurse is aware that contraindications to bethanechol include: a bladder atony. b peptic ulcer. c urinary retention. d hypothyroidism.

b peptic ulcer.

1 A patient is scheduled for colorectal surgery tomorrow. He does not have sepsis, his WBC count is normal, he has no fever, and he is otherwise in good health. However, there is an order to administer an antibiotic on call before he goes to surgery. The nurse knows that the rationale for this antibiotic order is to a provide empiric therapy. b provide prophylactic therapy. c treat for a superinfection. d reduce the number of resistant organisms.

b provide prophylactic therapy.

3 The nurse is assessing a patient who has been taking quinidine and asks about adverse effects. An adverse effect associated with the use of this drug includes: a muscle pain. b tinnitus. c chest pain. d excessive thirst.

b tinnitus.

10. 10.ID: 10068745705 What are the expected adverse effects of anticholinergic (parasympatholytic) drugs? (Select all that apply.) a. Diarrhea b. Dry mouth c. Dilated pupils d. Urinary retention e. Increased sweating

b. Dry mouth Correct c. Dilated pupils Correct d. Urinary retention Correct Anticholinergic drugs block the effects of the parasympathetic nervous system, producing sympathetic nervous system effects. These include mydriasis (dilated pupils), decreased bladder contraction, and decreased oral secretions. The effect on the GI system would be to decrease GI motility, not cause diarrhea. Sweating would decrease as a result of anticholinergic drugs.

12. 12.ID: 10068762504 For patients prescribed amiodarone (Cordarone), the nurse should monitor for which potential adverse effects of this drug? (Select all that apply.) a. Diarrhea b. Visual halos c. Hypothyroidism d. Photosensitivity e. Overgrowth of gum tissue f. Blue gray skin discoloration

b. Visual halos Correct c. Hypothyroidism Correct d. Photosensitivity Correct f. Blue gray skin discoloration Correct Potential adverse effects from amiodarone include visual halos, photosensitivity, photophobia, dry eyes, bluish skin discoloration, hyperthyroidism, hypothyroidism, constipation, and decreased libido.

2 Which statement is appropriate for the nurse to include in patient teaching regarding type 2 diabetes? a "Insulin injections are never used with type 2 diabetes." b "You don't need to measure your blood glucose levels because you are not taking insulin injections." c "A person with type 2 diabetes still has functioning beta cells in his or her pancreas." d "Patients with type 2 diabetes usually have better control over their diabetes than those with type 1 diabetes."

c "A person with type 2 diabetes still has functioning beta cells in his or her pancreas."

3 When taking a patient's drug history, the nurse asks about use of over-the-counter drugs. The patient responds by saying, "Oh, I frequently take aspirin for my headaches, but I didn't mention it because aspirin is nonprescription." What is the best response from the nurse? a "That's true, over-the-counter drugs are generally not harmful." b "Aspirin is one of the safest drugs out there." c "Although aspirin is over the counter, it's still important to know why you take it, how much you take, and how often." d "We need you to be honest about the drugs you are taking. Are there any others that you haven't told us about?"

c "Although aspirin is over the counter, it's still important to know why you take it, how much you take, and how often."

5 A patient is about to undergo cardioversion, and the nurse is reviewing the procedure and explaining moderate sedation. The patient asks, "I am afraid of feeling it when they shock me?" What is the nurse's best response? a "You won't receive enough of a shock to feel anything." b "You will feel the shock but you won't remember any of the pain." c "The medications you receive will reduce any pain and help you not to remember the procedure." d "They will give you enough pain medication to prevent you from feeling it."

c "The medications you receive will reduce any pain and help you not to remember the procedure."

2 The family of a patient who has recently been diagnosed with Alzheimer's disease is asking about the new drug prescribed to treat this disease. The patient's wife says, "I'm so excited that there are drugs that can cure this disease! I can't wait for him to start treatment." Which reply from the nurse is appropriate? a "The sooner he starts the medicine, the sooner it can have this effect." b "These effects won't be seen for a few months." c "These drugs do not cure Alzheimer's disease. Let's talk about what the physician said to expect with this drug therapy." d "His response to this drug therapy will depend on how far along he is in the disease process."

c "These drugs do not cure Alzheimer's disease. Let's talk about what the physician said to expect with this drug therapy."

4 A patient calls the family practice office to report that he has seen his pills in his stools when he has a bowel movement. How will the nurse respond? a "The pills are not being digested properly. You need to take them on an empty stomach." b "The pills are not being digested properly. You need to take them with food." c "What you are seeing is the waxy matrix that contained the medication, but the drug has been absorbed." d "This indicates that you are not tolerating this medication and will need to switch to a different form."

c "What you are seeing is the waxy matrix that contained the medication, but the drug has been absorbed."

5 Several patients have standard orders for acetaminophen as needed for pain. When the nurse reviews their histories and assessments, the nurse discovers that one of the patients has a contraindication to acetaminophen therapy. Which patient should receive an alternate medication? a A patient with a fever of 103.4° F (39.7° C) b A patient admitted with deep vein thrombosis c A patient admitted with severe hepatitis d A patient who had abdominal surgery 1 week earlier

c A patient admitted with severe hepatitis

2 A patient is receiving an opioid via a PCA pump as part of his postoperative pain management program. During rounds, the nurse finds him unresponsive, with respirations of 8 breaths/min and blood pressure of 102/58 mm Hg. After stopping the opioid infusion, what should the nurse do next? a Notify the charge nurse b Draw arterial blood gases c Administer an opiate antagonist per standing orders d Perform a thorough assessment, including mental status examination

c Administer an opiate antagonist per standing orders

3 For a patient receiving a vasoactive drug such as intravenous dopamine, which action by the nurse is most appropriate? a Monitor the gravity drip infusion closely, and adjust as needed. b Assess the patient's cardiac function by checking the radial pulse. c Assess the intravenous site hourly for possible infiltration. d Administer the drug by intravenous boluses according to the patient's blood pressure.

c Assess the intravenous site hourly for possible infiltration.

3 The nurse providing teaching for a patient who has a new prescription for beta1 blockers will keep in mind that these drugs may result in which effect? a Tachycardia b Tachypnea c Bradycardia d Bradypnea

c Bradycardia

4 When a patient is being taught about the potential adverse effects of an ACE inhibitor, which of these effects should the nurse mention as possibly occurring when this drug is taken to treat hypertension? a Diarrhea b Nausea c Dry, nonproductive cough d Sedation

c Dry, nonproductive cough

4 A patient with type 2 diabetes is scheduled for magnetic resonance imaging (MRI) with contrast dye. The nurse reviews the orders and notices that the patient is receiving metformin (Glucophage). Which action by the nurse is appropriate? a Proceed with the MRI as scheduled. b Notify the radiology department that the patient is receiving metformin. c Expect to hold the metformin the day of the test and for 48 hours after the test is performed. d Call the prescriber regarding holding the metformin for 2 days before the MRI is performed.

c Expect to hold the metformin the day of the test and for 48 hours after the test is performed.

2 A nurse with adequate knowledge about the administration of intravenous nitroglycerin will recognize that which statement is correct? a The intravenous form is given by IV push injection. b Because the intravenous forms are short-lived, the dosing must be every 2 hours. c Intravenous nitroglycerin must be protected from exposure to light through use of special tubing. d Intravenous nitroglycerin can be given via gravity drip infusions.

c Intravenous nitroglycerin must be protected from exposure to light through use of special tubing.

3 Which point will the nurse emphasize to a patient who is taking an antilipemic medication in the "statin" class? a The drug needs to be taken on an empty stomach before meals. b A low-fat diet is not necessary while taking these medications. c It is important to report muscle pain immediately. d Improved cholesterol levels will be evident within 2 weeks.

c It is important to report muscle pain immediately.

4 A patient is being assessed before a newly ordered antilipemic medication is given. Which condition would be a potential contraindication? a Diabetes insipidus b Pulmonary fibrosis c Liver cirrhosis d Myocardial infarction

c Liver cirrhosis

2 An older adult has been given a benzodiazepine for sleep induction, but the night nurse noted that the patient was awake most of the night, watching television and reading in bed. The nurse documents that the patient has had which type of reaction to the medication? a Allergic b Teratogenic c Paradoxical d Idiopathic

c Paradoxical

4 A patient who is about to be given octreotide is also taking a diuretic, IV heparin, ciprofloxacin (Cipro), and an opioid as needed for pain. The nurse will monitor for what possible interaction? a Hypokalemia due to an interaction with the diuretic b Decreased anticoagulation due to an interaction with the heparin c Prolongation of the QT interval due to an interaction with ciprofloxacin d Increased sedation if the opioid is given

c Prolongation of the QT interval due to an interaction with ciprofloxacin

4 During a teaching session for a patient who will be receiving a new prescription for the LTRA montelukast (Singulair), the nurse will tell the patient that the drug has which therapeutic effect? a Improves the respiratory drive b Loosens and removes thickened secretions c Reduces inflammation in the airway d Stimulates immediate bronchodilation

c Reduces inflammation in the airway

3 When the nurse is checking the laboratory data for a patient taking spironolactone (Aldactone), which result would be a potential concern? a Serum sodium level of 140 mEq/L b Serum calcium level of 10.2 mg/dL c Serum potassium level of 5.8 mEq/L d Serum magnesium level of 2.0 mg/dL

c Serum potassium level of 5.8 mEq/L

3 After a patient has been treated for depression for 4 weeks, the nurse calls the patient to schedule a follow-up visit. What concern will the nurse assess for during the conversation with the patient? a Weakness b Hallucinations c Suicidal ideations d Difficulty with urination

c Suicidal ideations

2 When administering beta blockers, the nurse will follow which guideline for administration and monitoring? a The drug may be discontinued at any time. b Postural hypotension rarely occurs with this drug. c Tapering off the medication is necessary to prevent rebound hypertension. d The patient needs to stop taking the medication at once if he or she gains 3 to 4 pounds in a week.

c Tapering off the medication is necessary to prevent rebound hypertension.

6 Which considerations are important for the nurse to remember when administering a benzodiazepine as a sedative-hypnotic drug? (Select all that apply.) a These drugs are intended for long-term management of insomnia. b The drugs can be administered safely with other CNS depressants for insomnia. c The dose needs to be given about 1 hour before the patient's bedtime. d The drug is used as a first choice for treatment of sleeplessness. e The patient needs to be evaluated for the drowsiness that may occur the morning after a benzodiazepine is taken.

c The dose needs to be given about 1 hour before the patient's bedtime. e The patient needs to be evaluated for the drowsiness that may occur the morning after a benzodiazepine is taken.

2 A teenage patient is taking a tetracycline drug as part of treatment for severe acne. When the nurse teaches this patient about drug-related precautions, which is the most important information to convey? a When the acne clears up, the medication may be discontinued. b This medication needs to be taken with antacids to reduce GI upset. c The patient needs to use sunscreen or avoid exposure to sunlight, because this drug may cause photosensitivity. d The teeth should be observed closely for signs of mottling or other color changes.

c The patient needs to use sunscreen or avoid exposure to sunlight, because this drug may cause photosensitivity.

5 The nurse is reviewing the orders for a newly admitted patient and sees an order for edrophonium (Tensilon). The nurse expects that this drug is ordered for which reason? a To reduce symptoms and delay the onset of Alzheimer's disease b To treat the symptoms of myasthenia gravis c To aid in the diagnosis of myasthenia gravis d To reverse the effects of nondepolarizing neuromuscular blocking drugs after surgery

c To aid in the diagnosis of myasthenia gravis

1 A patient has a new prescription for transdermal nitroglycerin patches. The nurse teaches the patient that these patches are most appropriately used for which reason? a To relieve exertional angina b To prevent palpitations c To prevent the occurrence of angina d To stop an episode of angina

c To prevent the occurrence of angina

5 A woman who has been taking an antibiotic for a UTI calls the nurse practitioner to complain of severe vaginal itching. She has also noticed a thick, whitish vaginal discharge. The nurse practitioner suspects that a this is an expected response to antibiotic therapy. b the UTI has become worse instead of better. c a superinfection has developed. d the UTI is resistant to the antibiotic.

c a superinfection has developed.

3 The nurse monitoring a patient for a therapeutic response to oral antidiabetic drugs will look for a fewer episodes of diabetic ketoacidosis (DKA). b weight loss of 5 pounds. c hemoglobin A1C levels of less than 7%. d glucose levels of 150 mg/dL.

c hemoglobin A1C levels of less than 7%.

1 The nurse is reviewing dietary supplements and recalls that under the DSHEA, manufacturers of dietary supplements are required to a follow FDA standards for quality control. b prove efficacy and safety of dietary supplements. c identify the active ingredients on the label. d obtain FDA approval before the products are marketed.

c identify the active ingredients on the label.

2 During assessment of a patient who is receiving digoxin, the nurse monitors for findings that would indicate an increased possibility of toxicity, such as: a apical pulse rate of 62 beats/min. b digoxin level of 1.5 ng/mL. c serum potassium level of 2.0 mEq/L. d serum calcium level of 9.9 mEq/L.

c serum potassium level of 2.0 mEq/L.

2 When preparing an IV solution that contains potassium, the nurse knows that a contraindication to the potassium infusion would be a diarrhea. b serum sodium level of 145 mEq/L. c serum potassium level of 5.6 mEq/L. d dehydration.

c serum potassium level of 5.6 mEq/L.

4 To help with the insomnia associated with thyroid hormone replacement therapy, the nurse will teach the patient to a take half the dose at lunchtime and the other half 2 hours later. b use a sedative to assist with falling asleep. c take the dose upon awakening in the morning. d reduce the dosage as needed if sleep is impaired.

c take the dose upon awakening in the morning.

5 During the assessment of a patient about to receive a cholinergic-blocking drug, the nurse will determine whether the patient is taking any drugs that may potentially interact with the anticholinergic, including: a opioids, such as morphine sulfate. b antibiotics, such as penicillin. c tricyclic antidepressants, such as amitriptyline. d anticonvulsants, such as phenobarbital.

c tricyclic antidepressants, such as amitriptyline.

2 The pharmacy has called a patient to notify her that the current brand of thyroid replacement hormone is on back order. The patient calls the clinic to ask what to do. Which is the best response by the nurse? a "Go ahead and take the other brand that the pharmacy has available for now." b "You can stop the medication until your current brand is available." c "You can split the thyroid pills that you have left so that they will last longer." d "Let me ask your physician what needs to be done; we will need to watch how you do if you switch brands."

d "Let me ask your physician what needs to be done; we will need to watch how you do if you switch brands."

5 A patient with diabetes has a new prescription for a thiazide diuretic. Which statement will the nurse include when teaching the patient about the thiazide drug? a "There is nothing for you to be concerned about when you are taking the thiazide diuretic." b "Be sure to avoid foods that are high in potassium." c "You need to take the thiazide at night to avoid interactions with the diabetes medicine." d "Monitor your blood glucose level closely, because the thiazide diuretic may cause the levels to increase."

d "Monitor your blood glucose level closely, because the thiazide diuretic may cause the levels to increase."

3 During a patient teaching session about antiparkinson drugs, the nurse will include which statement? a "The drug will be stopped when tremors and weakness are relieved." b "If a dose is missed, take two doses to avoid significant decreases in blood levels." c "Be sure to notify your physician if your urine turns brownish-orange in color." d "Take care to change positions slowly to prevent falling due to a drop in blood pressure."

d "Take care to change positions slowly to prevent falling due to a drop in blood pressure."

4 A patient is receiving dobutamine for shock and is complaining of feeling more "skipping beats" than yesterday. What will the nurse do next? a Monitor for other signs of a therapeutic response to the drug. b Titrate the drug to a higher dose to reduce the palpitations. c Discontinue the dobutamine immediately. d Assess the patient's vital signs and cardiac rhythm.

d Assess the patient's vital signs and cardiac rhythm.

4 While assessing a patient with angina who is to start beta blocker therapy, the nurse is aware that the presence of which condition may be a problem if these drugs are used? a Hypertension b Essential tremors c Exertional angina d Asthma

d Asthma

2 The nurse is giving a cholinergic-blocking drug and will assess the patient for which contraindications to these drugs? a Chronic bronchitis b Peptic ulcer disease c Irritable bowel syndrome d Benign prostatic hyperplasia

d Benign prostatic hyperplasia

5 When monitoring for the therapeutic effects of intranasal desmopressin (DDAVP) in a patient who has diabetes insipidus, which assessment finding will the nurse look for as an indication that the medication therapy is successful? a Increased insulin levels b Decreased diarrhea c Improved nasal patency d Decreased thirst

d Decreased thirst

5 While monitoring a patient who is receiving an infusion of a crystalloid solution, the nurse will monitor for which potential problem? a Bradycardia b Hypotension c Decreased skin turgor d Fluid overload

d Fluid overload

4 The nurse is preparing an infusion for a patient who has a deficiency in clotting factors. Which type of infusion is most appropriate? a Albumin 5% b Packed RBCs c Whole blood d Fresh frozen plasma

d Fresh frozen plasma

3 The nurse is reviewing the medication list for a patient who will be starting therapy with somatropin. Which type of drug would raise a concern that needs to be addressed before the patient starts the somatropin? a Nonsteroidal antiinflammatory drug for arthritis b Antidepressant drug c Penicillin d Glucocorticoid

d Glucocorticoid

3 Which nursing diagnosis is possible for a patient who is now recovering after having been under general anesthesia for 3 to 4 hours during surgery? a Impaired urinary elimination related to the use of vasopressors as anesthetics b Increased cardiac output related to the effects of general anesthesia c Risk for falls related to decreased sensorium for 2 to 4 days postoperatively d Impaired gas exchange due to the CNS depressant effect of general anesthesia

d Impaired gas exchange due to the CNS depressant effect of general anesthesia

1 The nurse is reviewing the medications that have been ordered for a patient for whom a loop diuretic has just been prescribed. The loop diuretic may have a possible interaction with which of the following? a Vitamin D b warfarin c Penicillins d NSAIDs

d NSAIDs

5 A hospitalized patient is complaining of having difficulty sleeping. Which action will the nurse take first to address this problem? a Administer a sedative-hypnotic drug if ordered. b Offer tea made with the herbal preparation valerian. c Encourage the patient to exercise by walking up and down the halls a few times if tolerated. d Provide an environment that is restful, and reduce loud noises.

d Provide an environment that is restful, and reduce loud noises.

5 The nurse is administering lidocaine and considers which condition, if present in the patient, a caution for the use of this drug? a Tachycardia b Hypertension c Ventricular dysrhythmias d Renal dysfunction

d Renal dysfunction

2 Patient teaching for a patient receiving an MAOI would include instructions to the patient to avoid which food product? a Orange juice b Milk c Shrimp d Swiss cheese

d Swiss cheese

2 When educating patients about the safe use of herbal products, the nurse remembers to include which concept? a Herbal and over-the-counter products are approved by the FDA and under strict regulation. b Herbal products are tested for safety by the FDA and the U.S. Pharmacopeia. c No adverse effects are associated with these products because they are natural and may be purchased without a prescription. d Take the product with caution because labels may not contain reliable information.

d Take the product with caution because labels may not contain reliable information.

4 During patient education regarding an oral macrolide such as erythromycin, the nurse will include which information? a If GI upset occurs, the drug will have to be stopped. b The drug needs to be taken with an antacid to avoid GI problems. c The patient needs to take each dose with a sip of water. d The patient may take the drug with a small snack to reduce GI irritation.

d The patient may take the drug with a small snack to reduce GI irritation.

5 The nurse is assessing a patient who is receiving a milrinone infusion and checks the patient's cardiac rhythm on the heart monitor. What adverse cardiac effect is most likely to occur in a patient who is receiving intravenous milrinone? a Tachycardia b Bradycardia c Atrial fibrillation d Ventricular dysrhythmia

d Ventricular dysrhythmia

1 For best results when treating severe pain associated with pathologic spinal fractures related to metastatic bone cancer, the nurse should remember that the best type of dosage schedule is to administer the pain medication a as needed. b around the clock. c on schedule during waking hours only. d around the clock, with additional doses as needed for breakthrough pain.

d around the clock, with additional doses as needed for breakthrough pain.

2 A patient is suspected of having adrenocortical insufficiency. The nurse expects to administer which drug to aid in the diagnosis of this condition? a octreotide (Sandostatin) b vasopressin (Pitressin) c somatropin (Humatrope) d cosyntropin (Cortrosyn)

d cosyntropin (Cortrosyn)

5 The nurse is assessing a newly admitted patient who has a history of seizures. During the assessment, the patient has a generalized seizure that does not stop for several minutes. The nurse expects that which drug will be ordered for this condition? a valproic acid (Depakote) b neurontin (Gabapentin) c carbamazepine (Tegretol) d diazepam (Valium)

d diazepam (Valium)

4 A patient who has recently had a myocardial infarction (MI) has started therapy with a beta blocker. The nurse explains that the main purpose of the beta blocker for this patient is to a cause vasodilation of the coronary arteries. b prevent hypertension. c increase conduction through the SA node. d protect the heart from circulating catecholamines.

d protect the heart from circulating catecholamines.

1 The physician has requested "lidocaine with epinephrine." The nurse recognizes that the most important reason for adding epinephrine is that it a helps to calm the patient before the procedure. b minimizes the risk of an allergic reaction. c enhances the effect of the local lidocaine. d reduces bleeding in the surgical area.

d reduces bleeding in the surgical area.


Conjuntos de estudio relacionados

Salesforce.com ADM 201 Study Guide - Practice 1

View Set

A&P Chapter 19 Heart & Cardiovascular Function

View Set

4B - SHERPATH - CHAP. 43 - care of the pt with a integumentary disorder - 10/7/22

View Set

Honors Science Worksheet 5-56 Structure of a Typical Flower

View Set

BJU Economics Chapter 6 - Economic Systems (control civilizations)

View Set

Radha Appan ISQS 4348 Exam 1 review

View Set

Combo with "AP Euro/H. World History Unit 14 Quiz" and 27 others

View Set

Chapter 5: Understanding Techniques for hiding and Scrambling Information

View Set